SPHR STUDY QUESTIONS

Lakukan tugas rumah & ujian kamu dengan baik sekarang menggunakan Quizwiz!

The large auto parts manufacturer for which you work has recently purchased land in an adjacent city and plans to build an additional warehouse. This is the best example of which of the following? a) A brownfield operation b) A greenfield operation c) A workforce plan d) An acquisition

B

A clothing company gives its employees time off work at a shelter for homeless women and children. It also donates clothing and shoes to the shelter. What phase of involvement does the company exhibit? A. reactive philanthropy B. strategic contributions C. mainstream involvement D. corporate accountability

C. The company's community involvement is linked to operational business concerns. Involvement consists of more than just giving money; it also involves the donation of goods and services.

What is the primary way a human capital management plan may be effective? a) If it aligns with the company's strategic plans b) If it properly forecasts the talent necessary to achieve company goals c) If it creates a competitive advantage for organizations d) All of the above

D

Which of the following describes an employee involvement strategy which requires organizational commitment at all levels of the organization, especially management, for it to be effective? a) Participative management b) Alternate work schedules c) Employee suggestion systems d) Teams

A

Which of the following processes is the LEAST likely to be used in a non-union environment? a) Compulsory arbitration b) Mediation c) Open door policy d) Peer review

A

What is a grievance?

A contract almost always includes this formal process for resolving differences of opinion

What is an injunction?

A court order that prohibits a party, employer or union from a specific course of action

What is a strike?

A refusal by union employees to work

The typical functions of management in a small business include all of the following a. Accounting b. Marketing c. Information Technology d. Strategic Planning

A, B & C. Business functions include operations, marketing and sales, finance and accounting, information technology and human resources.

All the following may cause job-related stress: (3 answers) A. job pressure B. self-efficacy C. workload D. conflict

A, C & D.

Which of the following statements about alternative dispute resolution (ADR) is true? A. it does not preclude litigation B. it requires an external third party C. it is used only in union environments D. it requires a jury of peers to hear the dispute

A. ADR does not prevent disputes from winding up in court or in formal arbitration, but it can result in fewer cases going to trial. It is generally used in union-free organizations. While some ADR systems do include peer review, it is not a requirement. Similarly, some systems may bring in neutral third parties from an outside company, but that is also not an ADR requirement.

The process of adjusting salary data to keep pace with market movement is referred to as A. aging B. leveling C. benchmarking D. compressing

A. Aging is the practice of adjusting outdated salary survey data and making it current. For example, assume pay increases average 3% a year. If we use data that is a year old, we need to increase the salary by 3%.

An OSHA inspector may enter an employers' worksite and conduct an inspection: A. Any time there is reasonable belief an OSH Act regulation has been violated B. Only when there is probable cause of a safety hazard C. Only if they have a search warrant D. Only if a worker reported an unsafe condition

A.

Balance sheets represent: a. The financial position of the organization. b. The financial position of the organization and level of revenue. c. An explanation of revenues, expenses, and profits over time. d. Gross profits.

A.

Which of the following companies might benefit most from a temp-to-hire program? A. A company that has had difficulty in finding and keeping a receptionist B. A company that does not have sufficient HR staff to administer payroll and benefits programs C. A company whose business fluctuates from season to season D. A company that needs highly technical workers for one- to two-year assignments

A. A company that has difficulty finding staff might benefit most from a temp-to-hire program. In this case, the company can try out an employee, offering a position only when there is a good match between employee and job. A company that does not have sufficient staff to administer HR programs might try leasing or outsourcing arrangements, while companies whose business fluctuates from season to season are probably best-off hiring "casual" workers who will work only during peak season.

An agreement between an employer and the union to waive the preelection hearing is called a A. consent election B. directed election C. voter eligibility election D. waiver election

A. A consent election is an agreement between an employer and the union to waive the preelection hearing. The elections that result from formal, as opposed to consent, procedures are called directed elections.

Which of the following is in the line of sight for computer programmers at a software development firm? A. reduce time to market for software products B. reduce defects in CD-ROM production C. improve communication with customers D. strengthen brand image in the marketplace

A. A goal that is the line of sign is a goal that employees an influence. They can see the direct result of their actions on attainment of the goal. A programmer can reduce time to market for software products by developing code models and programming tools that speed programming efforts. However, improved communication, brand image, or customer satisfaction are goals upon which the programmer will have little direct impact.

Which of the following is a written summary of the work to be done? A. job description B. job specification C. job context D. job ranking

A. A job description is a written description of the job and its requirements. A job specification is a written statement of the necessary qualifications of the job incumbent.

Identity and right to work can be verified by a A. US Passport B. voter's registration card C. US Social Security Card D. US citizen ID card

A. A voter's registration card demonstrates only identity. The US Social Security card and US citizen ID card demonstrate only the right to work.

Which of the following managed care plans will not cover any services provided outside of the network? A. EPO B. PPO C. POS D. PPA

A. An EPO (exclusive provider organization) requires participants use providers in the network or no payment will be made. The PPO (preferred provider organization) and PPA (preferred provider arrangement) permit access to treatment outside the network, but there may be additional costs. The POS (point-of-service) organization is a combination of a PPO and PPA and permits access to specialists.

An employee arrives at work a half hour early, makes coffee for herself, and reads the morning paper at her desk until it is time to start work. Must the employee be paid for this time? A. No, since the employee is not doing work B. No, as long as the employee leaves work one-half hour early C. Yes, since the employee is at her desk D. Yes, unless the employee has permission to come in early

A. An employee may come in early and wait for the workday to begin. As long as the employee is not doing work, the employer does not have to pay the employee. However, if the employee does even casual work--such as helping another employee or returning e-mail messages--the employee must be paid.

An organization that places more emphasis on training and formalizes policies and procedures is in what phase of its organizational evolution? A. Maturity B. Growth C. Introduction D. Decline

A. An organization that has time to formalize policies and procedures and uses training to maintain the flexibility and skills of its workers is in the maturity phase of the organizational life cycle.

After an employee removes a machine guard to keep the machine from jamming, he cuts his finger and requires medical treatment. This is an example of A. unsafe act B. unsafe condition C. unrecognized hazard D. unsafe work assignment

A. An unsafe act is an improper action taken by an employee that results in an incident. Dressing improperly or removing safety devices are examples. An unsafe condition is a hazardous circumstance that is likely to lead to an incident, such as poor ventilation, unsafe floor surfaces, and excessive work hours.

OSHA is primarily responsible for establishing workplace safety standards and: a. Conducting workplace inspections b. Conducting safety training classes c. Holding safety violation hearings d. Recommending safety related legislation

A. Occupational Safety and Health Act of 1970 has two primary administrative functions: establish safety standards and conduct workplace inspections

What is the difference between career management and career planning? A. career management focuses on the organization's needs; career planning focuses on the individual needs B. career management assesses career paths within the organization; career planning focuses on building a pool of workers qualified for specific assignments C. career management assumes that employees are responsible for planning their own careers; career planning assumes that the company is responsible for planning employee careers D. career management is based on communication between managers and employees; career planning is done by the employee alone

A. Career management focuses on the organization and concerns itself with identifying future staffing needs, matching organizational needs with individual abilities, and designing appropriate career development programs and paths. Career planning focuses on the individual, who assesses personal abilities and interests, evaluates career paths, and plans personal career goals.

Which of the following will be MOST effective in protecting proprietary information? A. confidentiality and nondisclosure agreements B. prohibition of employee-installed software C. installation of surveillance cameras D. banning of internet and e-mail access for employees

A. Confidentiality and nondisclosure agreements will be most effective in protecting proprietary information. Banning Internet and e-mail access is impractical, and surveillance cameras would have a minimal impact. Prohibiting software installations is more for the protection of the computer network than for protection of proprietary information.

The three current HR roles are: a. Administrative, operational, and strategic b. Recordkeeping, staffing, and training c. Administrative, strategic, and global d. Operational, legal compliance, and strategic

A. Current HR roles include administrative (recordkeeping and legal compliance), operational (concern with how the organization is run), and strategic (involved in long term goals of the organization). Historic HR roles include advisory (assist line managers with HR issues), service (assist employees with employment issues) and control (implement policies, procedures and rules).

Who are not protected under the Americans with Disabilities Act? A. Current illegal drug users B. Recovering illegal drug users C. Current legal drug users D. Current alcoholics

A. Current illegal drug users are not covered under Americans with Disabilities Act. Those who are covered include recovering illegal drug users, current legal drug users, and alcoholics.

Which of the following is considered direct compensation? A. cash achievement award B. use of a company car C. disability insurance D. deferred pay

A. Direct compensation deals with pay systems and includes cash achievement awards. Indirect compensation deals with benefits programs and includes items like a company car, disability insurance, and deferred pay.

Which of the following is an example of disparate treatment? A. Members of a protected group are subject to stricter attendance rules B. A neutral staffing practice results in discrimination against protected groups C. Height restrictions are set for all security guards D. All employees are required to take an intelligence test

A. Disparate treatment occurs when an employee who is a member of a protected group is treated differently from other employees; disparate impact occurs when a neutral practice results in unintentional discrimination for a protected group.

What is the appropriate bargaining unit (ABU)?

NLRB determines this based on "community of interest" factors and issues of proximity

The ADA applies to A. employers with 15 or more employees B. employers with 25 or more employees C. employers who contract with the federal government D. all employers regardless of the size of the company

A. Employers with 15 or more employees must follow ADA provisions

3. The engineering approach to workplace health and safety focuses on: A. the physical environment B. behavioral issues C. work schedules D. risk management

A. Engineering approach focuses on the physical environment, equipment, ergonomics, health issues, and hazard communication standard. The behavioral approach is focused on individual factors. The organizational approach looks at shift work and overtime.

Employers may test workers coordination each day before allowing them to work on dangerous equipment. This type of test is called: a. Fitness for duty testing b. Drug testing c. Coordination testing d. Productivity testing

A. Fitness for duty testing

A neutral employer who performs work that strikers would normally perform is an example of A. ally doctrine B. alter ego doctrine C. single employer doctrine D. joint employer doctrine

A. If a neutral employer performs work that striking worker would normally perform, the impact of the strike is moderated. The struck employer effectively uses the employees of its ally as a strike breaker. In this case, a union may extend its picketing to the neutral employer.

Starting in January, an employee sets aside $100 a month in a flexible spending account. The employee files a $1,200 medical claim in February. If the employee leaves the firm in March, the employee is A. entitled to full reimbursement of $1,200 B. entitled to only the amount of money paid into the account C. able to pay into the account after employment is terminated D. not entitled to any reimbursement

A. If qualified nonreimbursed expensed are incurred prior to the termination of employment, they are covered. (However, this rule does not apply to dependent-care reimbursement accounts). The benefit is no longer in effect once employment is terminated.

The ADA does NOT protect an employee who A. uses illegal drugs B. has epilepsy C. has AIDS D. has a serious psychological problems

A. Illegal drug users are not protected by the ADA. The other disabilities listed above are covered under the act.

Individual factors that may lead to increased workplace accidents include all the following EXCEPT: a. Job Design b. Boredom c. Stress d. Risk

A. Individual factors cause accidents. Lack of attention (boredom, repetitive duties, workplace stress, family stress). Employee attitude (risk takers and sabotage)

An employer may have a leave policy stating that: A. WC, FMLA, and any company provided PTO leave must run concurrently; B. WC, FMLA, and any company provided PTO must run consecutively; C. WC and FMLA may run concurrently, but workers must have the right to select when they will use PTO benefits; D. WC and PTO must be taken concurrently, but employees have the right to determine whether or not to request a FMLA leave.

A. It is an employer right to combine workers' compensation, Family and Medical Leave (FMLA), and other paid-time-off company provided leave

All of the following statements are appropriate to include in an employee handbook EXCEPT A. all regular employees will receive equitable severance pay in the event of separation B. the organization is an equal opportunity employer and will not tolerate discrimination C. standard hours of business operation are 8 AM to 5 PM PST D. we offer employees and visitors a smoke-free environment

A. It is best to play things safe and avoid putting specifics about severance pay in writing. Doing so makes it legally defensible for a disgruntled departing employee, who could easily hold the company to the terms as stated. All other examples are typically covered in an employee handbook.

A job design strategy that increases the variety of responsibilities but requires the same skill level is referred to as A. job enlargement B. job enrichment C. job simplification D. job specialization

A. Job enlargement attempts to alleviate the boredom and low morale associated with excessive job simplification. It increases the number of tasks to be performed, expanding the scope of the job and allowing for more variety, but all of the tasks require the same skill level. Job enrichment, which is sometimes confused with job enlargement, increases the depth of a job by adding increased responsibility for planning, organizing, controlling, and evaluation. It may improve morale but may not necessarily improve productivity. Job simplification and specialization generally reduce the number of tasks required of an employee.

Employees are given a chance to indicate an interest in an announced position through A. job posting B. skill tracking C. succession planning D. job analysis

A. Job posting gives employees the chance to respond to announcements of positions. Skill tracking, succession planning, and job analysis are human resource activities that are not based upon an employee's stated interest in a position.

Which of the following theorists defines quality as fitness to use, which emphasizes the reliability of a product or service? A. Juran B. Deming C. Crosby D. Baldrige

A. Juran defines quality as fitness to use

Which of the following statements about long-term protection is generally true? A. it is integrated with Social Security benefits B. it expires after 26 weeks of disability C. it becomes effective once an employee's sick-leave benefits expire D. it covers both work- and nonwork-related injuries

A. Long-term coverage is usually integrated with Social Security to avoid duplication of disability coverage. B is incorrect because there is no set number of weeks for coverage; it can begin at an early age and last until age 65. C is incorrect because long-term coverage generally does not become effective until an employee has exhausted both sick leave and short-term coverage. D is incorrect because work-related injuries are covered under workers' compensation.

Employers must keep a record of most work-related injuries and illnesses on a(n): a. OSHA Form 300 b. OSHA Form 300A c. OSHA Form 301 d. Form created by the organization

A. OSHA Form 300 is a Log of Work-Related Injuries and Illnesses

A consulting company recently changed its vacation and leave policies and wants to know how employees feel about the changes. Its 500 employees are located in 40 locations throughout the US and western Europe. Which of the following survey methods is likely to yield the highest response rate and best data? A. online survey B. telephone survey C. fax survey D. mail survey

A. Online surveys generally yeild the highest response rate due to the convenient 24/7 access via the internet or company intranet. In addition, responses to open-ended questions are often more complete if employees can type them in as opposed to writing them on a form. Given the time zone changes, phone interviews would be cumbersome; fax and mail surveys run the risk of being lost and not reaching all employees.

Which of the following is the best way for an organization to determine recruiting effectiveness? A. analyze yeild ratios B. conduct a needs assessment C. conduct employee surveys D. review industry data

A. Organizations need to evaluate recruitment strategies against objective criteria. Yeild ratios produce factual data that can be used to determine which source, method, or recruiter produces the greatest yeild.

What is the BEST way to gain attention for HR issues with Congress? a. Participate in Grass Roots efforts b. Lobby in Washington with other HR professionals c. Write your Senator's staff concerning recommendations d. Attend meetings to become more aware of issues facing Congress

A. Participate in Grass Roots Initiatives. Become involved in local HR organizations, stay informed on proposed legislation, write to congressional representatives, and if possible, speak to representatives.

For private-sector nonexempt employees, how must overtime be paid? A. in cash at a rate of one and one-half times their regular rate of pay B. in compensatory time off at a rate of one and one-half hours for every overtime hour C. In cash or compensatory time, as determined by the employee D. In cash or compensatory time, as determined by the employer

A. Private-sector nonexempt employees must be paid one and one-half times their regular rate of pay for hours worked in excess of 40 hours in any workweek. Public-sector employers may grant compensatory time off instead of cash.

Which of the following statements about recognitional picketing is true? A. it is limited to 30 days, at which time the union must petition of an election B. it is illegal if an election has been held within the past 24 hours C. it is legal if a certified union exists, but the majority of workers are dissatisfied with the union D. there are not recognitional picketing as long as no laws are broken

A. Recognitional picketing is used to gain recognition of the union as the employees' bargaining representative. It is limited by law to 30 days, at which the time the union must petition for an election or cease picketing.

An organization analyzes an assembly-line job, determines the most efficient assembly process, and institutes this process throughout the organization. This is an example of A. standardization B. specialization C. job rotation D. division of labor

A. Standardization focuses on creating processes that are intuitionalized and lead to reduced training time and costs greater job efficiency. Specialization occurs when people complete the same task, becoming more proficient over time. Division of labor reduces jobs to their smallest components and assigns a worker to each component. Job rotation is a strategy what shifts people between comparable but different jobs.

Strategic planning starts with an organization's A. mission statement B. environmental scan C. human resources inventory D. needs assessment

A. Strategic planning begins with a mission statement that articulates who the organization is, what the organization does, and where the company is headed. The mission sets the direction for all other activities. Without this direction, other activities such as needs assessments, inventories, and environmental scans have no direction. Therefore, the first step is to have the board or owners articulate an organization's vision.

Selection is the process of A. hiring the most suitable candidate for a position B. identifying the top applicants for a position C. hiring the best-qualified candidate for the position D. finding additional minority applicants

A. Selection does not necessarily mean hiring the best-qualified candidate for a position because that may preclude existing alternative action (i.e. hiring a qualified candidate from a protected class)

Which of the following is true of severance packages? A. they are not required by federal law B. they ensure that the employees will not file suit against the company C. they require employees to sign a waiver that they will not sue the company D. they will increase an employer's unemployment tax contributions

A. Severance packages are offered at the discretion of the employer. Federal law does not require them, although some states may have laws regarding severance. While they may diminish the chance that an employee will file suit, there is no guarantee. Offering severance packages does not increase an employer's unemployment taxes; this is an outcome of laying off workers whether or not severance packages are added.

A field HR unit that reports directly to corporate headquarters is referred to as what type of structure? A. Departmental B. Functional C. Matrix D. Divisional

B. A functional organization is more centralized, and its departments are arranged by function (HR, finance, marketing, etc.), with each department reporting to a regional or corporate headquarters.

A supervisor conducts an investigatory interview to obtain information that could be basis for disciplinary action. According to the Weingarten case, union employees may have which of the following attend the meeting? A. a union representative B. a member of management C. a friend or relative D. an attorney

A. The Weingarten case gave union workers the right to request the presence of a union representative at an investigatory meeting--a meeting whose purpose is to gather facts. Friends, relatives, or an attorney may not be present. These rights apply only to an investigatory interview, no tot meetings that impose disciplinary action. In the past, nonunion workers were also granted Weingarten rights, but court rulings took away these rights for nonunion workers.

An employer's best defense when a case of negligent hiring goes to court is A. produce evidence of employment and reference checking B. produce the employee's signed application form C. produce letters of reference provided by the employee D. prove that any misconduct occurred after regular work hours

A. The basic theory is that employers have a duty to protect workers from coworkers who the employer knew (or should have known) posed a risk. Proof that reference and employment checking has been done is the employer's best defense. A signed application form or letters of reference provided by the employee may not be accurate or present a complete picture. The fact that misconduct may have occurred after work hours does not relieve the employer from the obligation to protect coworkers from employees who pose a risk.

An employee is a 25-year veteran with the company. During that time, the employee has received numerous employee bonuses and awards and was offered another position by a competitor. The employee is given a pay raise and assured taht there will "always be job for you here." Management changes at the company, and six months later the employee is terminated. Which of the following claims by the employee will be most effective? A. the employer entered into an oral contract with the employee B. the employer is guilty of age discrimination C. the employer must grant a just cause for exception to employment-at-will D. the employer must offer a severance package to the employee

A. The best course of action by the employee may be to prove that an express oral contract existed and that he provided important consideration by not accepting the competitor's offer of employment. Such contract need not to be in writing, and the contract prevents the employer form claiming an employment-at-will relationship.

According to the Fair Credit Reporting Act, if an employer uses a credit report to deny a job, promotion, the employee must A. receive a pre-adverse action disclosure B. agree not to sue the company C. provide verbal authorization for the project D. agree that the information in the report is accurate

A. When denying a promotion based on the credit report findings, an employer must provide a pre-adverse action disclosure that includes a copy of the report and a summary of the employee rights prepared by the Federal Trade Commission

Which leadership theory maintains that performance depends on the interaction between situation favorableness and leadership style? A. contingency B. trait C. behavioral D. situational

A. The contingency theory suggests that group performance depends on the interaction between leadership style and situation favorableness, which is determined by leader-member relations, task structure, and position power.

Which of the following is NOT a goal of orientation? A. to provide remedial skills training B. to establish relationships with coworkers C. to understand company policies and procedures D. to gain a feeling of belonging

A. The goal of orientation is to smooth the way for new employees, making them less likely to leave the organization during the first year. Orientation introduces people to the company's culture and policies and helps them fit into the organization. Orientation does not provide the specific skills training that people need to perform effectively on the job.

In trend and ratio analysis, the key to accurately projecting past figures into the future depends on the A. relationships between two variables and whether they will stay the same B. length of time it takes to complete the analysis C. acceptance of the analysis by the organization D. support of senior management

A. The key to accurate projections is whether the relationships will continue to hold. The length of time it takes to complete the analysis could affect the relationships, but it is the relationship, not the time, that is the key factor. The acceptance and support of key management has nothing to do with the relationships.

According to situational theorists, which of the following leadership style is the most effective in managing unskilled, entry-level workers? a) Selling b) Telling c) Participating d) Delegating

B

Which of the following measures of central tendency refers to the value that occurs most frequently in a distribution? A. mode B. median C. mean D. range

A. The mode occurs most frequently in the distribution. The point at which the number of values below is equal to the number above; the mean reflects the average; and the range refers to the difference between the highest and lowest values.

Which of the following is the MOST important step in creating a strong safety culture? A. involving employees and top management in the creation of safety policies B. holding the line organization responsible for safety C. providing employee incentives for meeting safety goals D. allowing workers to file safety complaints

A. The most important step in the commitment of employees and top management. While the other actions are necessary, they are not enough if there is no policy on which management and employees agree.

An organization decides to develop a corporate citizenship program that gives employees time off to volunteer at a homeless shelter. A company task force creates a list of goals that the program hopes to accomplish. In which management function is the task force engaged? A. Planning B. Organizing C. Directing D. Controlling

A. The planning function involves activities such as setting goals, forecasting, and determining possible actions. The other management functions take place after the planning has been completed. The organizing function involves creating structures processes to help meet the goals, while the directing function involves engaging in activities necessary to help meet the goals. Finally, the controlling function involves monitoring the program to measure how well it is meeting the company's goals.

A supervisor counsels an employee about frequent tardiness. In spite of discussions, the employee is late for work twice the following week. What should be the next step in the disciplinary process? A. give the employee an oral warning B. give the employee a written warning C. give the employee time off without pay D. terminate the employee

A. The supervisor has already counseled the employee and discussed reasons for tardiness and possible solutions. According to the progressive disciplinary process, the next step should be a verbal warning. If the offense were more serious, some of the steps in the process could be skipped, but this appears to be a less serious offense.

Which of the following is considered direct taxable compensation? A. tips B. employee discounts on employer goods C. tuition reimbursement for job-related undergraduate credits D. business use of a company vehicle

A. Tips are considered taxable income

The BEST way to align HRD with organizational goals is for HR A. T professionals to A. participate in the strategic planning process B. involve managers in the development of training activities C. reduce learning time for mastery of critical skills D. increase the budget for training activities

A. To support organizational goals, HR professionals need input form a variety of sources: the business climate, upper management, and key stakeholders. The best link to these sources is through the strategic planning process.

The ability of an instrument to measure what is intended to measure is referred to as A. validity B. internal consistency C. reliability D. parallelism

A. Validity answers the question "What does the instrument measure?"

Which of the following explains why employers care responsible for the discriminatory acts of their supervisor? A. vicarious liability B. hostile environment C. quid pro quo D. reverse discrimination

A. Vicarious liability is the legal doctrine under which a party can be held liable for the wrongful actions of another party. Because of this doctrine, employers are legally responsible for the discriminatory acts of their employees.

Union recognition is traditionally a result of A. an NLRB-ordered election B. an NLRB order resulting from an employer C. voluntary recognition by the employer D. a management count of authorization cards

A. While all of the other options may lead to union recognition, recognition most commonly comes as a result of a secret ballot in an NLRB-conducted election.

Centralization of authority works best in organizations where A. economies of scare are important B. operating units need to collaborate C. employee turnover is high D. decision-making ability is expected of all employees

A. centralization is the degree to which decision-making authority is restricted to higher levels of management. When economies of scare are important, a single decision-making authority can best guarantee the consistency of results.

Which of the following takes both an internal and external snapshot of the organization? A. SWOT analysis B. Environmental scan C. Human resources inventory D. Human resource audit

A. the SWOT analysis looks at the organization's internal strengths and weaknesses and external opportunities and threats. The environmental scan looks only at the external conditions, while the human resources inventory and the audit are internally focused.

Which of the following statements about the Taft-Hartley Act is true? A. it allowed employers to file under unfair labor practice charges against unions B. it established the NLRB to encourage growth of the union environment C. it prohibited paycheck deduction of union dues D. it allowed employers to establish company-sponsored unions

A. the Taft-Hartley Act established a balance of power between union and management by designating certain union activities as unfair labor practices. B is incorrect because the NLRA established the NRLB to encourage union growth. C is incorrect because the Taft-Hartley Act allows deduction of union dues with the employee's written consent. D is incorrect because of the Taft-Hartley Act outlawed sweetheart contracts, forbidding employers from establishing company-sponsored labor unions.

What is a threat of reprisal?

An action that intimidates or coerces employees to act in a certain way

What is a consent election?

An agreement between an employer and the union to waive the pre-election hearing

What is job bidding?

An employee application system used to help employees change jobs

It is NOT considered insubordination to refuse an unsafe assignment as long as all of the following conditions are met: (3 answers) a. The employee "clocked out" b. The fear was objective and reasonable c. The employee notified the supervisor of the hazard d. The situation could not be corrected

B, C, D OSHA worker rights state it is not insubordination if: Fear was objective and reasonable, the employee notified the employer of the hazard, the situation could not be corrected or the employer refused to correct it. Every worker has the right to: work in a safe and healthy job site, refuse an unsafe work assignment, file a complaint with OSHA, request an inspection, and accompany the OSHA inspector

An organization determines the demographic composition of its workforce by jobs that have similar content, responsibilities, wage rates, and opportunity for advancement. This is a(n) A. organizational profile B. job group analysis C. availability analysis D. workforce analysis

B. A job group analysis lists all job titles that comprise each job group. Jobs are grouped according to whether they have similar content and responsibilities, wage rates, and opportunities for advancement.

What is the purpose of the Generally Accepted Accounting Principles (GAAP)? a) To create the body of knowledge for accounting professionals b) To guide the competencies required of certified public accountants c) To establish consequences for accounting professionals who behave in an unlawful manner d) To create checks and balances within accounting departments

D

Under WARN Act, what type of notice must an employer provide? A. 30 days' notice if 500 full- and part-time employees at one site will be laid off B. 60 days' notice if 33% of full-time employees at one site will be laid off C. 60 days' notice if 500 full- and part-time employees will be laid off D. 90 days' notice if a layoff is considered permanent

B. A mass layoff (33% of the full-time employees at a work site or 500 or more full-time employees) requires 60 days' notice.

Which of the following is a benefit funded by the Social Security Administration? A. paid sick time B. retirement plans C. Medicare D. life insurance

The correct answer is C. Medicare is funded by the Social Security Administration.

Which of the following is NOT regulated by the FLSA? A. exempt status B. employee benefits C. minimum wage D. overtime pay

B. Employee benefits are not governed by the FLSA. Commonly referred to as the Wage and Hour Law, the FLSA governs employee status, child labor, minimum wage, overtime pay, and record keeping

Which of the following examples does not describe a potentially violent employee? a. An employee is again turned down for a transfer to the first shift b. A worker cries often at work since her mother died last month c. One worker is very angry since a co-worker refused to date him based on rumors other workers told her about him d. An employee truly believes his supervisor is always giving him the worst assignments

B.

Which of the following safety concerns became a top priority for employers after the terrorist attacks of 9/11. a. Workplace violence b. Physical security c. Internet security d. Applicant screening

B.

Which job evaluation method is commonly used in the government and the public sector? A. ranking B. classification C. point-factor method D. factor comparison

B. A classification system groups jobs into a predetermined number of grades or classifications, each having a class description used for job comparisons. The best-known system is the General Schedule (GS) system used by the federal government.

An employee files a discrimination charge and is laid off a short time alter due to a downturn in the company's business. In this situation, the employer could be accused of A. sexual harassment B. retaliatory discharge C. involuntary retirement D. constructive discharge

B. A discharge is retaliatory if an employer punishes an employee found in engaging in a protected activity such as filing a discrimination charge or whistleblowing. A case could be mad taht the employee's discharge was a result of filing a discrimination charge.

A bonus paid to an employee based upon a management judgement made at the end of a time period is referred to as A. performance-based B. discretionary C. formula-based D. service-based

B. A discretionary bonus is given at the discretion of the manager. A performance-based bonus is measured against predetermined objectives, a formula-based objective is typically based on a percentage of profits, and a service-based bonus is based on tenure.

What is the value of a strong corporate culture? A. it makes maintenance of the status quo more likely B. it gives members an organizational identity C. it eliminates the need for a corporate ethics officer D. it makes it easier for employees to question corporate values

B. A strong corporate culture usually provides a sense of identity for employees and a commonality of values and goals that leads to effective change management and problem solving. Stronger cultures are more open to change and usually outperform those organizations with weaker cultures. On the negative side, a strong corporate culture may prevent employees from expressing differences of opinion.

Which of the following statements about a good training objective is true? A. it includes the consequences of not meeting the objective B. to states what a trainee will be able to do as a result of training C. it states the role that management will play in the training D. it includes directions for meeting the objective

B. A training objective defines the behavioral outcomes participants can expect from the training experience. The training is built around the objectives and provides the direction necessary to meet the objectives

Which of the following learning curves is typically associated with complex tasks such as problem solving? A. plateau B. S-shaped C. increasing returns D. decreasing returns

B. An S-shaped curve is a combination of increasing and decreasing returns and is common when a trainee learns a difficult task that also requires insight. Learning may be slow at the onset and then steadily increase as the process becomes more familiar. Learning to diagnose and repair a piece of equipment is an example.

A job function is usually considered essential if it A. can be performed by many employees B. requires highly specialized skills C. is performed infrequently D. can be completed quickly

B. An essential job function is one that is performed regularly and requires highly specialized skills or expertise, and, in many cases, the reason the job exists is to perform the function. Nonessential functions can be performed by many people in a department. If not completed, there are often minimal consequences.

Which of the following tests measures the capacity to learn or acquire new skills? A. achievement B. aptitude C. psychomotor D. personality

B. Aptitude tests measure a person's capacity to learn or acquire new skills. Achievement tests measure what a person knows or can do

A company has 1,000 eligible employees in a prospective bargaining unit. How many of these employees must sign an authorization card before the NLRB will order an election? A. 200 B. 300 C. 500 D. 501

B. At least 30% of the eligible employees must sign authorization cards.

A training specialist is able to shoe that training has decreased turnover by 10%. At what level has the training been evaluated? A. learning B. results C. reaction D. behavior

B. At the results level, the effect of training on organizational goals is demonstrated. Results such as productivity, turnover, quality, time, and costs are concrete examples of training results. At the reaction level, trainees evaluate how well they liked the training experience. At the learning level, trainees are tested to demonstrate how well they learned fact and concepts. At the behavior level, trainees are observed to determine whether they are using what they have learned on the job.

Which of the following is true of a benchmark job? A. the job is in the middle of the pay range for its class B. the job is used in the external labor market for setting wages C. the job content changes frequently and is unique to an employer D. the job is entry-level in its class

B. Benchmark jobs are well known and understood by most employers. They are generic positions that are common throughout almost all industries and therefore used in setting wages.

The leadership theory that classifies leaders according to their concerns with people and production was developed by A. Hersey-Blanchard B. Blake-Mouton C. Covey D. McClelland

B. Blake-Mouton's theory suggests that the most effective leaders are equally concerned with people and production to the maximum degree. Hershey-Blanchard's theory espouses situational leadership that requires a leader to change leadership styles based on a situation. To Covey, leadership deals with vision and the ability to keep the organization's mission in sight while producing results. McClelland focuses on the characteristics of people with high needs for achievement, therefore defining what type of person might become a leader.

Which of the following situations that lead to workplace violence can be controlled by an organization? A. unstable economy B. pressure for increased productivity C. domestic problems D. low self-esteem

B. Conditions causing employee frustration and anger can lead to violence. Examples include pressure for productivity, rigid management style, and layoffs.

Union decertification provides a mechanism for A. employers to bring unfair labor practice charges against unions B. employees to terminate a union that negotiates an ineffective contract C. unions to give up their right to represent a bargaining unit D. employees to remove a union's authority to enforce union security clauses

B. Decertification gives union members the right to terminate a union that it believes is not representing its best interests. At least 30% of bargaining unit members must sign the petition for decertification. Deauthorization is often confused with decertification; deauthorization involves filing a petition to withdraw the union's authority to maintain security provisions such as union shop clause.

The Drug-Free Workplace Act, 1988, requires federal contractors to: A. restrict smoking in the workplace B. establish drug awareness programs C. establish EAP to eliminate illegal drug use in the workplace D. test all employees for illegal drug use

B. Drug-Free Workplaces Act requires federal contractor with contracts of $100,000 or more to agree to provide drug-free workplaces. It encourages other employers to do the same.

Employers calculate incidence rates for injuries to determine A. employees who need safety training B. the company's standing in the industry C. the frequency of OSHA inspections D. work sites that need to be carefully monitored

B. Employers calculate incidence rate statistics for injuries and illnesses per 100 full-time employees for number of lost work days per 100 employees. These statistics can be used to determine where the company stands in the industry (nationwide or statewide) and how the company compares to the Standardized Industrial Classification (SIC). The statistics are not kept to single out employees who need safety training or particular work sites that need monitoring or to predict the frequency of OSHA inspections.

Which of the following could be an exception to the employment-at-will concept? A. an employee is fired for engaging in misconduct B. an employee is terminated for filing workers' compensation benefits C. an employer and employee agree to server the employment relationship D. an employer terminates an employee who does not have an employment contract

B. Employment-at-will is the concept that an employer or employee may sever the employment relationship at any given time for any reason unless prohibited by law or an employment contract. There are a number of exceptions to the employment-at-will, including public policy and just cause. Filing for workers' compensation benefits falls into the public policy exception that states employees cannot be fired for fulfilling legal obligations for pursuing certain statutory rights. In MOST states, filing workers' compensation benefits falls into this category.

Which of the following is true of experimental designs? A. subjects are not assigned randomly to a group B. only the experimental group receives the treatment C. it is the least rigorous type of design d. a control group is not needed

B. Experimental designs are the most rigorous types of designs. Tue experiments utilize a control group.

An organization that gives preference to applicants referred by current employees could be guilty of A. disparate treatment B. a historically discriminatory practice C. reverse discrimination D. intentional discrimination

B. Giving preference to applicants referred by current employees may perpetuate past polices that were discriminatory. It may serve to maintain the current racial or ethnic mix, making it hard for members of some groups to get hired. The organization is not guilty of intentional discrimination because it does not intentionally treat or evaluate protected groups differently from other employees.

A warehouse distribution organization has been concentrating on coaching and developing employees. The manager of the distribution department adjusts his leadership style to what is necessary for each employee, depending on the employee's level of job development and maturity. The manager of the marketing department does not change his leadership style but rather changes the factors surrounding a situation to manage the marketing department employees. Which leadership theory is the distribution manager following? A. Blake-Mouton's managerial theory B. Hersey-Blanchard's situational theory C. Feidler's contingency theory D. Herzberg's motivational theory

B. Hersey-Blanchard's theory states that the leadership style is situational, based upon the employee's maturity. As the employee matures, the leadership style should become more relationship-motivated than task-motivated.

OSHA gives employees the right to A. file an unfair labor practice charge B. request an OSHA investigation C. override employer safety procedures D. refuse to wear safety equipment

B. If employees believe an unsafe situation exists, they have the right to request an OSHA investigation. However, they are obligated to follow the employer's safety procedures, wear the proper protective equipment, and go through the proper channels in raising safety issues. OSHA does not deal with unfair labor practices.

The change process goes more smoothly when: A. senior management announces changes to the employees B. employees have input to change alternatives C. HR is responsible for the change initiative D. change is critical to the organization's survival

B. If possible, employees should be given the opportunity to discuss the change options. This makes it more likely that they will accept whatever decision is made. At the very least, it helps change agents in the organization understand employee issues and reactions and plan for ways to deal with them.

Which pay system is best suited to routine jobs where worker qualifications increase over time? A. productivity-based B. time-based C. single-rate D. performance-based

B. In a time-based system, the rate is based on longevity in the job and pay increases occur according to a predetermined schedule. A time-based system recognizes that an employee's skills increase so does his/her value.

A pregnant employee comes into contact with chemicals as part of her job and occasionally suffers from minor skin rashes. In this situation, the employer must A. transfer the employee to a job where there are no toxic substances B. inform the employee of the hazards associated with the job C. require the employee take special protection measures D. require the employee to get a doctor's permission to continue working

B. In the Johnson Controls case (1990), the Supreme Court ruled that decisions about the welfare of future children must be left to their parents rather than to employers who hire their parents. As a result, most sex-specific fetal policies that bar fertile women from work requiring contact with hazardous substances violate Title VII. However, the employer may minimize potential liability by fully informing female employees of the hazards of the job.

Of the four elements associated with marketing, which includes supply chain management? A. product B. place C. price D. promotion

B. In the marketing world, place refers to distribution. Placing the product requires marketers to think about how the product gets from the manufacturer to the customer. This process is known as supply chain management

Which of the following is true about a cash balance plan? A. money deposited into the account has already been taxed B. benefits are not affected by decreases in the fund C. employees assume the investment risk of the plan D. employees are required to take a lifetime annuity at retirement

B. Increases or decreases to the fund do not directly affect the benefit promised to the employee, and the employer assumes the risks and rewards on the plan assets. At retirement, employees may take a lump-sum distribution or a lifetime annuity. Money in the plan is tax-deferred, and employees can do a lump-sum rollover into an IRA or a plan taht accepts rollovers.

With what type of budget are prior budget items used as the basis for new funding? a. Zero-based b. Incremental c. Formula d. Gross profit

B. Incremental-prior budget items are the basis for new funding. Zero-based-new budgets start at zero. Formula-overall cost applied to calculate increases. Operating- focus on income statement. Capital-plan to finance long-term fixed assets.

A project manager who works to build team norms and foster harmony is assuming which responsibility? A. Informational B. Interpersonal C. Conceptual D. Decisional

B. Interpersonal responsibilities include serving as a leader and a liaison. Top responsibilities include building team norms and fostering harmony.

An employee works in the engineering department of a manufacturing company and has a consulting contract with one of the company's clients. Which ethical problem might exist? A. insider trading B. conflict of interest C. kickbacks D. copyright violation

B. It is conflict of interest for an employee to have a consulting with one of the company's clients or competitors. In this case, the employee may be taking business away from the company or suggesting strategies that may or may not be in the best interest of the company.

The three key elements included in a job analysis are A. responsibilities, tasks, reporting structure B. knowledge, skills, abilities C. competencies, qualifications, procedures D. reporting structure, pay range, essential job functions

B. Job analysis focuses on the key human requirements necessary to perform the job. It does not deal with organizational structures, pay ranges, or procedural issues.

Exempt status would include a manager who A. makes less than $455 per week B. directs five people and has hiring and firing responsibility C. receives overtime pay during a particularly busy work period D. is docked for less than one full day away from the job

B. Managers who manage at least two or more employees and have hiring and firing authority qualify for the executive exemption

Which of the following OSHA standards is known as the Employee Right-to-Know Law? A. Occupational Noise Exposure B. Control of Hazardous Energy C. Personal Protective Equipment D. Hazard Communication

D. The Hazard Communication standard requires that employees be informed of hazardous substances in the workplace through labeling, training, and written hazard communication programs.

For an employee to qualify for workers compensation benefits, he/she must: a. Demonstrate the employer's responsibility for the accident. b. Have incurred an injury/illness that took place or was aggravated by the work environment. c. Have permanent partial disability. d. Have been injured on employer's premises.

B. Must be work-related

Which of the following does NOT impact the number of pay grades within an organization? A. size of the organization B. organization's pay position in the marketplace C. vertical distance between highest- and lowest-level jobs D. organization's pay increase and promotion polices

B. Pay grades are used to group together jobs having approximately the same internal worth. Ther are no fixed rules as to how many pay grades a company should have. Factors such as the size of the organization, the vertical distance between the highest- and lowest-level jobs, and the company's pay policies after affect the number of pay grades. The organization's pay position in the marketplace does not affect the number of pay grades

Which of the following compensates employees who arrive at work but find that no work is available? A. on-call pay B. reporting pay C. premium pay D. travel pay

B. Reporting pay compensates workers for whom no work is available. On-call pay compensates employees who may be called in but are not actually working. Premium pay is provided when employees work on a vacation day or holiday. Travel pay compensates nonexempt workers for time they spend traveling to work assignments.

When a qualified white male is denied an opportunity because preference is given to a member of a protected group, it is known as A. reasonable accommodation B. reverse discrimination C. quota system D. undue hardship

B. Reverse discrimination occurs when a qualified person in a nonprotected class is denied an opportunity in favor of a member of the protected class who may be less qualified. Quota systems set aside a specific share of openings for minorities or women. The Supreme Court has ruled against strict quota systems in a number of cases.

Task significance is the extent to which a job A. offers workers freedom and autonomy B. is meaningful and important to others C. requires completion of a "whole" unit of work D. requires clear and direct feedback

B. Task significance is the extent to which a job has substantial impact on other people and therefore becomes more meaningful. The remaining options refer to other design characteristics of jobs: A refers to autonomy; C refers to task identity; and D refers to feedback

The process of analyzing exposure to risk and determining how to best handle such exposure is: A. terrorism prevention B. risk management C. risk aversion D. risk prevention

B. Risk management is responsible for: analyzing organization's exposure to risk and determine how to handle such exposure

The management practice designed to protect the physical well being of workers is: A. health management B. safety management C. security management D. workers compensation

B. Safety management is concerned with the physical protection or workers. Heal management is concerned with workers' overall health. Security management protects organization's facilities and employees

Which of the following is a retirement savings plan for employees of certain tax-exempt organizations such as public sector schools and hospitals? A. 457 plans B. 403b plans C. money purchase plans D. section 125 plans

B. Section 403b of the Internal Revenue Code creates retirement savings plans for certain tax-exempt organizations such as K-12 public schools, colleges, universities, hospitals, libraries, philanthropic organizations, and churches.

What is the level of violation if the employer knew, or should have known, of a substantial probability that death or serious physical harm could result from the unsafe working condition? A. A criminal offense B. A serious violation C. A felony D. A willful and repeated violation

B. Serious violations could probably cause death or serious physical harm. Intermittent danger is the most serious and requires immediate correction. Other than serious could affect safety, not likely to cause death or serious harm. De miminis is no immediate connection between violation and employee health or safety. Willful and repeated the employer does not correct, and penalty can be severe if worker dies.

Which of the following is true of short-term objectives? A. They create direction and synergy for the organization B. They establish priorities for allocating resources C. They must be created before unit strategies can be developed D. They take the place of action plans

B. Short-term objectives specify the milestones that must be reached before long-term goals can be met. The short-term objectives are focused and specific, and at this point the organization can allocate the resources to meet them. It is important to note that short-term objectives do not take the place of action plans, which are direct outgrowth of short-term objectives

What was the Norris-LaGuardia Act?

No arbitrary injunctions and no "yellow dog" contracts

According to the Equal Pay Act, employers must A. reduce the pay of employees to equalize pay between the sexes B. pay women and men the same pay for the same work C. pay the rates set by union contracts even if there are inequities D. pay similarly for jobs requiring comparable skills and knowledge

B. The Equal Pay Act attempts to prohibit wage discrimination by requiring equal pay for equal work. The law was aimed at remedying the proactive of paying women lower wages than men for essentially the same work. Equal work should not be confused with the concept of comparable work, which is based on similar pay for comparable jobs.

OSHA's General Duty Clause requires that A. employees report unsafe working conditions to OSHA B. employers provide hazard-free work environments C. employers develop standards if none exist for their industries D. employees need not report for duty if their safety is at risk

B. The General Duty Clause of the Occupational Safety and Health Act requires that employers provide a work environment free of hazards, even if there are no standards governing the work area or industry.

An HR manager is directly involved in an employment decision that leads to an EEOC complaint. Legal counsel prepares an answer to the complaint and files it with the court. As the case proceeds, the manager will NOT be asked to A. respond to requests for information and documents B. prepare other HR managers and witnesses for deposition C. sit with legal counsel during the trial D. attend the deposition of the plaintiff

B. The HR manager will be deposed but will not be responsible for preparing other witnesses. This is the responsibility of legal counsel. Counsel will probably ask the HR manager to attend the deposition of the plaintiff since the manager can provide insights into the testimony. For the same reason, it is likely that the manager will sit with counsel during the trial. During all phases of the proceedings, the HR manager will be asked to provide information or documents.

Who determines whether a proposed bargaining unit is appropriate? A. mediator B. NLRB C. union leaders D. employer

B. The NLRB determines whether a proposed unit is appropriate in a given situation and whether the employees have a sufficient common interest so as not to create substantial conflicts in bargaining or representation.

Which of the following statements about the Privacy Act of 1974 is true? A. it establishes concept of just cause B. it protects federal employees from unauthorized disclosure of information C. it establishes the concept of due process D. it lists all federal laws pertaining to record keeping

B. The act gives federal-sector employee the right to know the type of information collected on them, review their files, have incorrect information removed, and restrict distribution of information.

Which of the following is a primary advantage of the balanced scorecard? A. it separates business strategy from day-to-day activities B. it relates the performance of the business functions to the organization's mission C. it focuses on financial measurements and hard data results D. it tracks progress against goals without raising accountability issues

B. The balanced scorecard links business strategies with day-to-day activities and aligns business function measurements with organizational strategies. As a result, an organization can track progress, reinforce accountability, and prioritize improvement opportunities. It includes four related perspectives (finance, customers, internal processes, and learning and growth) rather than just a financial perspective.

Which of the following court cases established the criteria for disparate impact? A. Washington V Davis B. Griggs V Duke Power C. Albemarle Paper V Moody D. McDonnell Douglas Corp V Green

B. The case of Griggs v. Duke Power recognized disparate impact and established that it is not necessarily enough to show a lack of discriminatory intent. Washington v. Davis recognized that even if a selection tool had an adverse impact, as long as it was a valid prediction of job-related success, it could be used. Albemarle Paper v. Moody ruled that tests used for promotion or selection must be valid predictors for job success. McDonnell Douglas Corp v. Green established the four criteria necessary for a prima facie case for disparate treatment.

An interviewer believes that the older workers are more difficult to train and make more mistakes than younger workers. This form of performance appraisal error is A. deficiency B. contamination C. bias D. conflict

C. Bias occurs when an appraiser's values, beliefs, or prejudices distort ratings. For example, a manager may believe that female employees with young children should not work outside the home and will rate employees in this situation more critically.

An employee files a grievance with the immediate supervisor claiming that a contract violation has occurred. The union steward agrees. What is the next step in the process? A. the employee testifies in front of a grievance committee B. a department head and higher-level union official discuss the grievance C. the grievance is settled by a third-party, neutral arbitrator D. legal counsel from the union and the company meet to settle the grievance

B. The grievance is written and reviewed by an intermediate manager or department head and a higher-level union official. At this stage, the grieving employee is not present and is represented by the union. The grievance must be settled within the time frame established by the contract, or it is elevated to the next level.

An employee who is good at answering customer questions on technical software is not good at completing paperwork or working with fellow employees. In spite of this, the employee is rated highly in all performance categories. What performance appraisal error has occurred? A. recency B. halo effect C. bias D. leniency

B. The halo effect occurs when an employee's excellent performance in one area causes the appraiser to also give the employee high ratings in all other areas.

Workers' compensation is regulated by the A. US Department of Labor B. states C. private insurance companies D. Social Security Administration

B. The individual states prescribe the rules governing coverage, eligibility, types of benefits, and the financing of benefits

A detailed step-by-step description of a company's customary method of handling activities is a A. policy B. procedure C. rule D. vision

B. The key phrase in this question is step-by-step. Only a procedure provides detailed, step-by-step methodology for handling activities. A policy is merely a statement reflecting an organization's philosophy, objectives, or standards; a rule states directly what must be done or avoided; and a vison articulates, in general terms, what an organization would like to achieve.

A qualified applicant who has performed a similar job in the past refused employment by a company because of a history of nervous breakdowns. Under Title I if the ADA, the company should A. hire the applicant but require a psychiatric exam B. hire the applicant since the applicant is qualified for the position C. refuse to hire the applicant because of the risk of negligent hiring D. refuse to hire the applicant for this position but find another position in the company

B. The law forbids employment discrimination against qualified individuals with a physical or mental disability. Since the applicant has the skill and experience to do the job and can perform the essential functions of the job, the applicant should be hired.

A small company does not have the right time or personnel to administer benefits, payroll, or personnel records programs. Which of the following flexible staffing alternatives might be an option for them? A. Hiring independent contractors instead of employees B. Leasing employees from a professional employer organization C. Using in-house temporary employees D. Instituting a temp-to-hire program

B. The most practical option is leasing. The company will transfer all of its employees to the professional employer organization (PEO), which assumes all of the HR functions and lease the employees back to the company. This arrangement is invisible to employees and eliminates the administrative burden for the company.

The PRIMARY purpose of an HR audit is to A. complete a cost-benefit analysis of an HRIS B. evaluate he effectiveness of the HR function C. determine HR staffing requirements D. determine which HR functions will be outsourced

B. The primary purpose of the HR audit is to evaluate the effectiveness of the HR function. The end result is not a cost-benefit analysis, a staffing plan, an outsourcing plan, or a proposal for a new HRIS, although these activities may be secondary results of the audit.

Which of the following activities is typical of a transactional leader? A. provides vision to workers B. intervenes when standards are not met C. gives personal attention D. communicates high expectations

B. The transactional leader offers the promise of reward or the threat of discipline, depending upon the followers' performance of measurable tasks. Intervening when standards are not met fits this description. The other activities listed are transformational, intended to motivate and inspire employees.

What is the weighted average of the following salaries? Salaries # of incumbents $20,000 2 $30,000 1 $35,000 2 $40,000 1 A. $25,000 B. $30,000 C. $31,250 D. $35,000

B. The weighted average takes into consideration the number or incumbents who receive each salary. Salaries # of incumbents Salaries Paid $20,000 2 $40,000 $30,000 1 $30,000 $35,000 2 $70,000 $40,000 1 $40,000 Totals 6 $180,000 180,000/6=$30,000

An organization received 150 applicants from an advertisement, 50 of whom were minority applicants. It interviewed 36 qualified applicants. The organization hired two applicants, one of whom is minority. What is the yeild ratio of qualified applicants to total applicants? A. 8% B. 24% C. 32% D. 50%

B. This yeild ration is determined by dividing the number of qualified applicants by the number of total applicants (36/150=24%)

HR can BEST assist in creating an ethical organization by A. handling all ethical issues that arise B. training leaders on ethical practices C. referring all ethical issues to legal counsel D. initiating a "tip line" for reporting ethical violations

B. To create an ethical organization, leaders and managers need to know how to recognize ethical dilemmas and how to deal with them. Leaders need to know what the organization values and what its expectations are for its leaders and employees.

A digital media company decides to replace the Dell computers employees currently use with the latest Apple laptops. If employees start to struggle with the new computers, which talent management strategy should HR recommend? A. training B. development C. discipline D. bonuses

The correct answer is A. Training is effective when the issue is knowledge-based or when employees lack a particular skill, such as becoming acquainted with a new computer.

Vroom's expectancy theory states that A. employees dislike rigid controls and want to accomplish something B. employees' effort is related to the likelihood of perceived success C. employees work to meet their physical and social needs D. employees are likely to quit their jobs if treated unfairly

B. Vroom's theory is based on a person's belief that a certain level of effort will lead to the expected result. A describes McGregor's Theory X and Theory Y; C describes Maslow's hierarchy-of-needs theory; and D describes the equity theory

An organization that offers brown-bag lunch meetings with tips on weight-reduction is probably doing it as a part of a: A. benefit plan B. wellness program C. EAP D. stress reduction initiative

B. Wellness programs are designed to keep employees healthy and productive.

A state's minimum wage is $7.50 in comparison to the federal minimum wage of $7.25. An employer in this state must pay employees A. $7.25 B. $7.50 C. an average of the two amounts D. any amount over $7.25

B. When state and federal minimum wage amounts differ, the employer must follow the regulation that most benefits the employee. Therefore, the employee must be paid $7.50/hour

The MOST basic reason employees join unions is to A. belong to a group with similar goals and ideals B. help guarantee a safe, secure work environment C. exercise their leadership abilities D. fulfill their needs for self-esteem

B. While all of the above options can be reasons for union membership, the most basic need, according to Maslow is the need for safety and security

Strikes that occur without the approval of union leadership are called A. jurisdictional strikes B. wildcat strikes C. sympathy strikes D. economic strikes

B. Wildcat strikes are work stoppages that are neither sanctioned nor stimulated by the union, although union officials may be aware of them. These strikes may also take the form of excessive absences, especially when there are no-strike clauses in contracts. Jurisdictional strikes are the result of disagreements between unions; they occur when one union's members walk out to force an employer to assign work to the instead of another union. Sympathy strikes occur when one union expresses its support for another union's strike, even though it has no dispute with the employer. Economic strikes are strikes that occur when collective bargaining fails to reach an agreement.

A company trainer is assigned to the task of creating an orientation handbook for newly hired employees. The copyright for the book belongs to A. the public domain B. the company C. the writer D. both the company and the writer

B. Work that is completed by an employee and is within the employees' regular job duties does not require a written assignment. The employer automatically is the copyright owner.

Analyzing the future hiring needs for an annual strategic plan is the best example of which of the following? a) The Delphi technique b) The nominal technique c) Forecasting d) Cost-benefit analysis

C

Employee Assistance Programs (EAPs) help employees with all types of problems except: A. personal problems B. illegal substance abuse issues C. work skills issues D. behavioral problems

C.

An MSDS contains which of the following? A. results of hearing exams B. lockout and tag out signs C. procedures for handling hazardous substances D. mine site inspection reports

C. A Material Safety Data Sheet (MSDS) contains procedures for handling hazardous substances

Which of the following is true of a balance sheet? A. it includes transactions without a definite monetary value B. it indicates the net income of the company C. it summarizes an organization's assets, liabilities, and equity D. it measures the difference between the cost to produce a product and the price for which it is sold

C. A balance sheet summarizes the financial position of a company at a given time. it lists the assets and liabilities of the organization and includes only transactions measurable in money.

An employee's son is no longer eligible for coverage under the company's health plan. According to COBRA regulations, the dependent is eligible for how many months of insurance continuation? A. 18 B. 29 C. 36 D. 48

C. A dependent who loses dependent status may continue benefits for 36 months

The Department of Labor schedules an audit that focuses on decision making that occurs in the C-suite. This type of audit is a A. compliance review B. desk audit C. glass ceiling audit D. compliance check

C. A glass ceiling audit focuses on review of programs at executive levels. It focuses primarily on the decision making of CEOs and senior executives and is generally a sensitive, private investigation.

An approach to bargaining in which both sides act reasonably and sincerely try to reach an agreement. A. bona fide qualifications B. mediation C. good faith D. accommodation

The correct answer is C. Good-faith bargaining involves a situation in which both sides act reasonably and sincerely try to reach an agreement.

An HR professional wants to use a diagram to represent the percentage of turnover for each of the last five years. Which quality tool will be most effective for visualizing turnover trends? A. check sheet B. control chart C. histogram D. pareto chart

C. A histogram uses a bar-graph chart to summarize key data. It works best for visualizing how change has occurred over time. The histogram makes it easy to see the data for each year and makes trends easy to spot.

Which of the following statements about job specifications is true? A. they summarize the mist important features of a job B. they describe the working conditions C. they list the qualifications necessary to perform the job D. they list all of the essential functions of the job

C. A job description is a written description of the job and its requirements. A job specification is a written statement of the necessary qualifications of the job incumbent. The job specification can be a section of the job description or a separate document.

A multinational corporation sends an international assignee to Brazil to improve processes in its manufacturing plant. The assignee receives the same wages as his Brazilian counterpart. What compensation approach is the company using? A. headquarters-based balance sheet B. home-country-based balance sheet C. pure localization D. better-of-home-or-host-country

C. A pure localization approach pays assignees the same compensation as it does local nationals in the same position. This can be relatively simple to administer when the assignee comes from a low-salary country, but it usually involves some allowances for assignees from high-salary countries

What is the first thing management should do when there is a workplace accident? A. notify OSHA B. develop training so it doesn't happen again C. investigate the cause of the accident D. make recommendations for future prevention

C. Accident investigations-root cause analysis: review scene, interview witnesses, prepare OSHA report if necessary, make recommendations.

Which of the following required workers who do not join a union to pay the equivalent of union dues? A. closed shop B. dues checkoff C. agency shop D. no-lockout

C. An agency shop clause states that even if workers do not join a union, they must still pay the equivalent of dues to the union. An agency shop clause may be prohibited by state law in right-to-work states. A closed shop clause states taht union membership is a condition of hiring; such a clause is illegal. Dues checkoff refers only to the mechanism by which dues can be collected. Under dues checkoff, the employee gives written authorization for the employer to deduct dues and initiation fees and remit them directly to the union. No-lockout is an agreement in which the company agrees not to lock out workers during a labor dispute for the life of the contract.

An employment application may include sections dealing with A. arrest records B. credit ratings C. authorization of information D. family status

C. An application form should ask applicants to verify that the information they have provided is correct. Asking for information related to arrest records, credit ratings, or family status may be discriminatory and should be avoided.

OSHA gives employers the right to A. disregard OSHA standards if company standards exist B. apply for a temporary variance if meeting an OSHA standard would increase costs C. refuse an OSHA inspection unless a search warrant is produced D. sue OSHA if they disagree with a citation and proposed penalty

C. An employer has the right to request a search warrant before an inspection can begin. A is incorrect because a company may not disregard OSHA standards if they have their own standards in place. However, the company can apply to OSHA for a permanent variance if they can prove that company facilities or procedures are at least as effective as those required by the standard. B is incorrect because a temporary variance can be requested only if the company does not have the materials, equipment, or personnel to come up to the standard within the required time. D is incorrect because the appropriate action for an employer is to file a Notice of Contest with OSHA Area Director within 15 days of a citation and proposed penalty.

An employer can require an applicant to take a polygraph test when the applicant will A. be in a position to learn confidential trade secrets B. work in a department that has been under investigation in the past C. have direct access to the distribution of controlled substances D. become a key member of the management group of the company

C. Applicants who will work with controlled substances can be required to take a polygraph test. A, B and D are not correct because employers may not use polygraph tests in instances like these to make employment decisions.

At which level of cognitive learning is an employee able to make judgements as to which manufacturing process produces better results? A. application B. analysis C. evaluation D. synthesis

C. At the evaluation level, a trainee can make judgements about which process, product, or solution is better than another.

Which of the following recruitment methods usually results in lower employee turnover? A. college placement offices B. advertisements C. employee referrals D. walk-ins

C. Because employees know the corporate culture and job specifications, they usually recommend people who fit in well. Therefore, these people may be more likely to remain with the organization and have the highest one-year survival rates.

The concept of broadbanding was developed to A. limit the autonomy of line managers B. reduce employee mobility within the organization C. work with flatter organizational structure D. provide narrower salary ranges

C. Broadbanding combines several salary grades or job classifications with narrow pay ranges into one band with a wider salary spread. This approach supports the de-layering efforts designed to reduce the number of reporting levels within an organization.

Which of the following is considered direct pay and immediately taxable? A. deferred profit-sharing plans B. Non leveraged ESOPs C. cash profit-sharing plans D. restricted stick grants

C. Cash profit-sharing plans provide payments in addition to the employee's normal rate of pay, based on the organization's profitability. These payments, as well as cash bonuses, are considered direct pay and are taxed accordingly.

Which of the following describes the difference between a committee and a task force? A. a committee is temporary and works outside of usual organizational boundaries; a task force is a permanent group dedicated to a specific task B. committee membership is voluntary, and committee members keep their normal jobs; task force members are appointed and are relieved of normal job duties C. a committee has an ongoing charter; a task force is temporary D. a committee is made up of employees from the same functional area; a task force is cross-functional

C. Committees, such as a safety committee, generally have an ongoing charter. While the members of the committee may change over time, the committee itself endures. On the other hand, a task force is disbanded once it has tackled the long-term strategic objective assigned to it.

Common law is based on A. EEOC directives B. congressional legislation C. court decisions D. torts

C. Common law is based on court decisions rather than codified laws. Statutory law is enacted by legislation and enforced by the EEOC. A tort is a wrongful act, the result of which may be a civil suit.

Which of the following is NOT a compensable factor? A. skill B. responsibility C. seniority D. working conditions

C. Compensable factors reflect how work is done and are supported by documentation such as job descriptions. The compensable factors flow from the work itself and not the employee holding the job. Therefore, seniority is not a compensable factor in determining the relative worth of a job.

The insurance that pays for employee's on-the-job injuries or illnesses is: a. Unemployment insurance b. FMLA protection c. Workers' Compensation insurance d. Occupational Safety and Health Insurance

C. Compensation is insurance for employee's on-the-job physical injury, stress related injury or illness, or environmentally caused illness

Which of the following statements describes public-sector labor relations? A. compulsory arbitration is common B. bargaining covers a wide range of mandatory issues C. the Taft-Hartley Act gives public-sector employees limited strike rights D. negotiators have full authority to agree to settlements

C. Compulsory arbitration, in which the law requires that both parties submit to arbitration, is common in the public sector. Negotiators are often restricted in their ability to make concessions, and, most often, strikes are prohibited.

Which of the following is an example of construct validity? A. assistants with college degrees are evaluated against those with no degrees B. flight controllers are tested for leadership and critical-thinking skills C. computer programmers are asked to debug a section of code D. new hires are tested, and their job performances are later compared to the test results

C. Construct validity is used to measure traits that are directly related to job performance. For flight controllers, the ability to think critically and lead others is key to performing effectively. It takes complex statistical analyses to determine that a test actually measures the job-related traits

Content validity is the extent to which a selection device measures A. differences between good performers and poor performers B. personality traits that are related to job performance C. knowledge, skills, and abilities used on the job D. experience as a predictor of job success

C. Content validity involves testing that determines if the employees can complete tasks that are part of the job. For example, a math test for a cashier tests the skills and knowledge needed to perform the job.

Which measure of association measures the relationship between two variables? A. range B. regression C. correlation D. deviation

C. Correlation measures the relationship between two variables. Regression involves the relationship between two or more variables and defines the ability to predict a variable based on knowledge about another variable.

A manufacturing company institutes an employee feedback group to improve relations with its workers. Which of the following actions might be perceived as a violation of the NLRA? A. the committee investigates safety problems and considers ways to make the workplace safer B. the committee certifies taht certain employees have reached higher skill level and should receive a pay increase C. the committee presents a proposal to management on increasing vacation time D. the committee stops the shipping of products that do not meet plant standards

C. Courts have found that committees that negotiate or present proposals to management for their consideration may actually be functioning as employer-dominated labor organizations. Any committee whose purpose is to "deal with" management runs this risk. On the other hand, courts have found that committees are legal when they exercise the authority delegated to them by management to operate within certain parameters. The kinds of activities described in options A, B and D are examples of legal activities and are comparable to those actions that would be taken by a first-line supervisor in a plant.

Decentralization works best in organizations where A. a uniform policy is important B. contracts are negotiated with outside agents C. a quick response to problems is desired D. employees do not want additional responsibility

C. Decentralization is the degree to which decision-making authority is given to lower levels in an organization's hierarchy. Pushing a decision down to its lowest authority level in an organization produces a quick response to problems.

A bill originates in a House committee and is forwarded to a subcommittee for review. The subcommittee conducts hearings on the bill and amends it. At this stage, the bill is sent to the A. full House for a final vote B. Senate for a floor debate C. president for his or her signature D. House committee for a vote

D. The House committee receives the bill back from the subcommittee and can conduct further hearings and vote on whether to recommend the bill to the House. This procedure is called "ordering a bill reported"

Which of the following situations illustrates a corporate strategy of differentiation? A. A company orders large quantities of parts to get the greatest discount B. The finished product is sold at a lower price than that of the competition C. A company develops a product that is uniquely easy to use and charges a premium price D. A company sells in volume and offers discounts to its best customers

C. Differentiation focuses on setting a product or service apart by giving it unique characteristics that consumers value and for which they will pay a premium.

Which of the following is generally true of EAPs? A. Employees must report their progress to a supervisor B. Supervisors may discuss their employee's progress directly with the counselor C. Employees may contact the EAP directly D. Employees who do not make progress are dropped from the program

C. Employees may make direct contact with the social service agency. They need not wait for a referral, and all communications that occur during counseling sessions are confidential.

Under the ADA, which of the following statements is true? A. preemployment medical examinations may be required before job offer is extended B. employers must set affirmative action plans for the disabled C. employers do not need to accommodate a disability it it results in undue hardship D. rehabilitated drug users are excluded by the law

C. Employers are not required to make accommodations that result in undue hardship. A is not correct because preemployment medical exams are prohibited except after an employment offer has been extended. B is incorrect because employers are not required to set affirmative action plans for the disabled. D is incorrect because the ADA's definition of disability includes rehabilitated drug users.

Which of the following statements about workers' compensation is true? A. it covers all worker's health problems B. it is funded by employers and regulated by the federal government C. it is paid even if an accident is the employee's fault D. it pays all injured workers the same benefits

C. Employers assume all costs of work-related injuries, regardless of who is to blame. A is incorrect because workers' compensation covers only those conditions that are caused, aggravated, precipitated or accelerated by work activity. B is incorrect because although workers' compensation is funded by the employer, it is regulated by the states, not the federal government. D is incorrect because compensation is tied to fixed schedules of minimum and maximum payments and is often based on the employee's earnings at the time of the injury and the nature of the impairment.

Which of the following questionnaire approaches elicits the broadest range of answers? A. rating scales B. yes/no answers C. essay questions D. multiple-choice questions

C. Essay questions are open-ended and allow respondents to provide as much input as they would like.

Child labor safety concerns are addressed by the: A. Occupational Safety and Health Act B. Americans with Disabilities Act C. Fair Labor Standards Act D. National Institute of Occupational Safety and Health

C. FLSA regulates the use of child labor. It sets restrictions for workers under the age of 18 (work times, working conditions and types of work)

A warehouse distribution organization has been concentrating on coaching and developing employees. The manager of the distribution department adjusts his leadership style to what is necessary for each employee, depending on the employee's level of job development and maturity. The manager of the marketing department does not change his leadership style but rather changes the factors surrounding a situation to manage the marketing department employees. Which leadership style is the marketing manager following? A. transformational leadership B. Blake-Mouton's managerial theory C. Fiedler's contingency theory D. Hersey-Blanchard's situational theory

C. Fred Feidler proposed that three factors determine the favorableness of the leadership environment; leader-member relations, task structure, and position power. The factors are identified from most important to least important. Together, they determine how favorable situations are for leaders.

Which of the following statements about single-rate pay system is true? A. it provides opportunity for progression within a grade B. it is typically used for exempt jobs C. it disregards performance or seniority D. it is not applicable in a union environment

C. In a single- or flat-rate systems, all employees are paid at the same rate of pay, regardless of performance or seniority. A is incorrect because a single-rate structure is not related to progression within a grade. B is incorrect because a single-rate structure is generally used for hourly or nonexempt jobs. D is incorrect because unions frequently prefer single-rate systems that are established during collective bargaining.

Which of the following actions BEST illustrates HR's strategic role in the organization? A. Institute radical changes in the organization B. Monitor and ensure compliance with change C. Influence and affect management's view on organizational change D. Deal with resistance to organizational change

C. In its strategic role, HR influences management's views on organizational change, helping management identify and deal with organizational change issues before any action is taken. Once senior management institutes the changes however, HR plays a secondary role in dealing with possible resistance and monitoring compliance.

In which of the following types of interviews is each applicant asked the same questions? A. patterned B. nondirective C. structured D. behavioral

C. In structured interviews (also called repetitive interviews) the interviewer asks every applicant the same questions

Common situs picketing occurs when A. commonly owned companies performing similar work have separate operations, one union and one nonunion B. a neutral employer is engaged in operations that are a phase of the struck employer's work C. a secondary employer who occupies common premises with the primary employer is affected by picketing D. unions distribute handbills urging customers to refuse to purchase products from the stuck employer

C. It describes common situs picketing, which is lawful if the picket signs clearly state the employer with whom the employees have a dispute. A is commonly referred to as double breasting, B refers to the ally doctrine, and D describes consumer picketing.

Which of the following is a nonquantitative method of job evaluation? A. factor comparison B. point-factor method C. job ranking D. job qualification

C. Job ranking is a nonquantitative method of evaluation that places different jobs in order without a numeric value being assigned. It shows that Job A is more important than Job B but not how much more important. The factor comparison and point-factor methods are quantitative methods of evaluation and involve the ranking of jobs using a point scale or weighing.

What is the advantage of nonqualified deferred compensation plans? A. they allow all company employees to contribute more than the limits prescribed by qualified plans B. they are not subject to ERISA and are protected from creditors C. they provide incentives for key management to stay with the organization D. they provide more favorable tax advantages to employers than qualified plans

C. Nonqualified deferred compensation plans allow organizations to provide additional benefits to a select group of key executives, which motivated them to say with the organization. A is incorrect because all company employees cannot patriciate; company-wide plans must be administered as qualified plans. B is not correct because although nonqualified plans are not subject to ERISA, the funds are not protected against bankruptcy, receivership, or creditors. D is incorrect because the tax ramifications for employers are less favorable than for qualified plans.

If an employer can show that complying with an OSHA standard would be more unsafe or dangerous than non-compliance, he may be granted a(n): a. Suspension b. Temporary variance c. Permanent variance d. Exception

C. Permanent variance may be requested if the current methods are safe or safer than the OSHA standard

OSHA's Form 300 Log of Work-Related Injuries must be completed: A. by every employer who has at least one employee B. within ten working day of an injury C. within eight hours of a work-related death D. by a person who observed the injury or illness

C. OSHA's Form 300 requires employers with 11 or more employees to complete the report for each occupational illness or injury within seven working days from the time the employer learns of it. A work-related death or hospitalization of three or more employees must be recorded and reported to the nearest OSHA office within eight hours.

According to OSHA, which of the following occupational injuries must be recorded? A. all disabilities, even if they do not result in any loss of work time B. all injuries, even those requiring minor first aid C. all injuries resulting in transfer to another job D. all injuries reported by workers, regardless of severity

C. Occupational injuries must be recorded if they result in death, disabilities that cause one or mor lost workdays, restriction of work or motion, loss of consciousness, transfer to another job, medical treatment other than minor first aid, significant injury or illness diagnosed by a physician, work-related case involving cancer, chronic irreversible disease, a fractured or cracked bone, or a punctured ear drum.

According to the ADEA, which of the following statements is true? A. high-level managers have no mandatory requirements for retirement B. a company may discontinue pension accruals for employees over the age of 65 C. employers may fire an employee over the age of 40 for a good cause D. employers do not need to offer insurance to employees covered by Medicare

C. Older workers, like other workers, may be fired for good cause, such as poor performance. A is incorrect because high-level managers can be required to retire at age 65 if they receive a company-sponsored retirement benefits of at least $44,000 per year. B. is incorrect because a company cannot limit or classify employees in any way that adversely affects their status. D. is incorrect because employers must offer employees age 65 and older the same insurance they offer younger workers.

All of the following are measures of productivity except: A. turnover rate B. revenue per employee C. workers' comp cost per employee D. cost of a new hire

The correct answer is A. Turnover rate measures the frequency with which employees leave the company; this is not directly related to productivity, which is essentially a function of comparing revenues (or units of production) against costs. All the other choices explicitly involve either revenues or costs to the company.

Which of the following is the MOST important priority at an accident scene? A. preserve the evidence B. keep management informed C. protect others from injury D. isolate witnesses

C. Protecting others from injury takes top priority. Secondary priorities include preserving evidence and keeping management informed. Holding witnesses together is one way to preserve evidence, but it is handled only after the other priorities are addressed.

A clerk is paid $13.50/hr when the pay range for the assigned grade is $12 to $13 an hour. This is referred to as a A. green-circle rate B. pay differential C. red-circle rate D. compressed salary

C. Red-circle rates are those in which the pay rate is above the range maximum

Which of the following is an example of reinforcement according to a fixed ratio? A. monthly paycheck B. occasional praise C. sales commission D. yearly review

C. Sales commission is a fixed-ratio reinforcer. The reinforcement is provided after a specific number of responses. Commission is linked to dollars sold, thus establishing a fixed ratio. A monthly paycheck or yearly review is a fixed-interval reinforcer, given after a specific interval of time passes. Occasional praise is a variable-ratio reinforcer, given after a random number of responses.

When paid union organizers infiltrate a company and begin organizing efforts, this process is known as A. leafleting B. organizational picketing C. salting D. campaigning

C. Salting occurs when unions hire and pay people to apply for jobs at companies targeted for unionization. When the employees are hired, they begin organizing efforts. If the employer fails to hire a salt or takes any adverse employment action, the salt may file an unfair labor practice charge against the company.

Which of the following laws restricts arbitrary injections against nonviolent union activity? A. Taft-Hartley Act B. Labor-Management Reporting and Disclosure Act C. Norris-LaGuardia Act D. Railway Labor Act

C. The Norris-LaGuardia Act restricts arbitrary injunctions against nonviolent union activity. Sweetheart contracts and union unfair labor practices were banned by the Taft-Hartley Act. Closed shop expectations for construction trades were allowed according to the Labor-Management Reporting and Disclosure Act. The Railway Labor Act gave railroad employees the right to organize and bargain collectively.

An organization is concerned that managers have too many employees reporting to them and too many employees who report to more than one manger, hindering efficiency and resulting in conflicting goals. Which type of OD strategy will address this problem? A. interpersonal B. technological C. structural D. process

C. Structural intervention strategies look at how the company is helped or hindered by its organizational structure. Interpersonal strategies deal with work relationships between employees, ad technological (process) strategies focus on activities such as work process flow and coordination among departments.

Which of the following gave union members the right to secret ballot elections for union offices and the right to sue the union? A. National Lator Relations Act B. Railway Labor Act C. Labor-Management Reporting and Disclosure Act D. Labro Management Relations Act

C. The Labor-Management Reporting and Disclosure Act was intended to protect employees from corrupt or discriminatory labor unions. It gave union members the right to secret ballot elections for union offices, protection from excessive dues, freedom of speech in the union matters, and the right to sue the union.

Which of the following is prohibited by the NLRA? A. arbitrary injunctions B. collective bargaining C. employer domination of unions D. yellow-dog contracts

C. The NLRA attempted to balance the interests of management and labor and prohibited the employer from unfair labor practices such as union domination. The Norris-LaGuardia Act prohibited arbitrary injunctions. The NLRA did not prohibit collective bargaining; it established mandatory subjects for collective bargaining. Yellow-dog contracts were prohibited by the Norris-LaGuardia Act.

Why should candidates who supply resumes also complete application forms? A. It is an EEOC requirement B. The resume may provide facts the employer is prohibited from requesting C. The forms require applicants to verify that the content is correct D. The forms provide information on fair employment practices

C. The application form asks the employee to sign and verify that the information is correct. It also states consequences, such as termination, if the information is found to be false. A is not correct because an application form is not an EEOC requirement. B is not correct because according to the EEOC and the courts, applications should not be constructed in such a way that they elicit discriminatory information. D is not correct because legally a form does not need to include information on fair employment practices.

Which of the following learning curves illustrates how a trainee learns the basics of a new, complex skill? A. plateau B. S-shaped C. increasing returns D. decreasing returns

C. The beginning of the curve is slow while the trainee learns the basics. Then performance takes off as skills and knowledge are acquired.

Employers are MOST likely to prevent substance abuse problems by A. implementing random drug testing of all employees B. searching employee workstations and company vehicles C. establishing written policies and providing training D. tracking the individual productivity of employees

C. The best prevention strategy is to establish clear and specific substance abuse policies, provide employee training, and follow through on policy enforcement. Legal and privacy issues make drug searches and random tests difficult. Although tracking of employee productivity can indicate changes in productivity, substance abuse may not be the cause of the changes

The burden of verifying that a new employee is eligible to work in the U.S. lies with the A. USICS B. Social Security Administration C. employer D. employee

C. The employer must fill out Form I-9, which verifies the employee's identity and right to work in the U.S. The employer keeps the form on file for at least three years or one year after the person leaves the place of employment.

A company that offers a defined benefit plan to its employees and funds the plan using a final-pay formula bases benefits on A. a set dollar amount for each year of service under the plan B. a percentage of the average earnings of the employee over the years of the plan C. average earnings for a specified number of years at the end of the employee's career D. a percentage of pay for each year of plan participation

C. The final-pay formula looks at the final years of a participant's career (usually five), based on the presumption that earnings will be the greatest then. A is incorrect because it describes a flat-dollar formula. B and D describe career-average formulas.

Replacement planning is based on an organization's ability to A. plan specific goals and assignments for individuals B. find candidates with development potential C. identify the best candidate for a job D. test candidates' potential early in their careers

C. The key phrase for a job makes C the correct answer. Replacement planning focuses on finding a person who could move into a key job in the organization. Planning specific goals and assignments for individuals, finding candidates with developmental potential, and testing candidates early in their careers are all hallmarks of succession planning.

At the end of the first day of a sales training seminar, a representative is able to list the five steps in the selling skills model. This is evidence of which level of learning? A. application B. analysis C. knowledge D. synthesis

C. The knowledge level requires a learner to recall specific facts. The learner is not yet able to interpret the information or apply it to work.

Which of the following statements regarding hot-cargo agreements is true? A. They are unlawful and regarded as unfair labor practices. B. They may only be undertaken in specific circumstances. C. They specify benefits to be paid to workers handling hazardous materials. D. They are commonly used in the shipping industry.

The correct answer is A. A hot-cargo agreement, also called a hot-cargo clause, is an agreement between a union and an employer that requires the employer to stop doing business with another organization. Such agreements are illegal.

What is the MOST important reason for conducting an internal marketing campaign for a new training program? A. it protects a company from charges of discrimination B. it makes HRD more visible to the organization C. it increases employee motivation to participate D. it increases the company's reputation in the marketplace

C. The most important reason to market a program internally is to get participants excited about learning new content. Training will not be effective if participants are not motivated to attend. While internal marketing may make HRD more visible and shows that the company is making an effort to include all employees, this is not the primary message of the marketing. As the marketing is internal, it is not designed to enhance the reputation of the company.

An organization that hires experienced personnel and is willing to exceed the standard pay range to quickly build its workforce is in what phase of its organizational evolution? A. Maturity B. Growth C. Introduction D. Decline

C. The organization is in the introduction phase and needs to do whatever is necessary to get its product or service off the ground. Therefore, the organization is willing to pay more to recruit a talented, already trained workforce. An organization at maturity will focus on controlling labor costs, while an organization that is in a growth stage will add new positions but will not exceed the standard pay range to do so. An organization in decline is likely to be downsizing or shrinking the workforce through attrition.

To qualify under ERISA, a pension plan must A. be explained verbally to employees B. allow special provisions for highly compensated executives C. include a vesting schedule D. allow all employees to participate

C. The plan must be in writing and communicate to employees, and it must contain a vesting schedule that lets employees know when they are eligible for accrued benefits. D is not correct since employees cannot contribute until they satisfy eligibility requirements.

A project manager would be more inclined to use a PERT chart rather than a Gantt chart when a manager needs to: A. assign particular tasks to team members B. monitor the exact time that each step of the project takes C. plan the details of the project D. show the critical path of a project

D. The PERT chart is best used to illustrate the length of a project, which is determined by adding up the time required on the critical path. The Gantt chart lists all activities and the start and stop date for each. Team members can easily see the start and end dates for each activity they are assigned and can track their time on task.

It is within the union's right to do which of the following? A. institute a secondary boycott against a neutral employer B. require employers to hire only union members C. dismiss grievances it does not think should be taken to arbitration D. require the employment of more workers than is necessary

C. The union may decide that a grievance should not be taken to arbitration. If the employer and the union agree, then grievance may be dropped. A is incorrect because the Labor-Management Relations Act made secondary boycotts (directed at a primary party by an action against a third party) illegal. B is incorrect because unions may not require union membership as a condition of employment; the closed show was outlawed by the Taft-Hartley Act. D is incorrect because featherbedding (requiring more workers than is necessary) is not within the rights of the union.

A Confined Space Entry standard requires employers to A. provide continuous and unobstructed exit from a building B. eliminate any confined spaces that may entrap workers C. classify dangerous confined spaces as permit-required D. provide on-site emergency rescue teams

C. This standard requires employers to determine which spaces are dangerous and to classify those spaces as "permit-required confined spaces." A written safe-entry program must be prepared and made available for review by employees, their representatives, and OSHA; however, on-site emergency rescue teams are not required. The Emergency Exit Procedures standard requires continuous and obstructed exit from a building.

An organization received 150 applicants from an advertisement, 50 of whom were minority applicants. It interviewed 36 qualified applicants. The organization hired two applicants, one of whom is minority. What is the yeild ration of minority applicants to total applicants? A. 12% B. 25% C. 33% D. 50%

C. This yeild ration is determined by dividing the number of minority applicants by the number of total applicants (50/150=33%)

The assignment of a level of probability for all types of losses to which an organization may be exposed is known as A. impact assessment B. risk containment C. vulnerability analysis D. loss-prevention planning

C. Vulnerability analysis measures the degree of probability that a loss will occur by assigning levels of probability for all types of losses to which the organization may be exposed

A company purchases new billing software that will be used by its customer service and accounting departments. Which of the following training methods will be MOST effective in training employees to use the new system? A. reading a PowerPoint presentation B. group discussion and structured exercises C. demonstration and structured exercises D. reading and case study

C. When learning new software, it is most important to see how the software works and practice using it. A demonstration of each of the software functions, followed by hands-on practice, is an effective way to teach employees the skills that are required on the job.

A form of interview bias in which strong candidates who interview after weak ones appear more qualified is known as A. cultural noise B. negative emphasis C. contrast effect D. halo effect

C. When strong candidates who interview after weak ones appear even more qualified than they actually are, it is called the contrast effect.

A company has high turnover in its telemarketing department. Part of the reason for the turnover is that employees deal with stressful situations and difficult people. Which of the following would provide the MOST realistic job preview? A. giving applicants a walking tour of the workplace B. allowing applicants to talk with employees from other departments C. allowing applicants to listen in on actual telemarketing calls D. watching videos of how to deal with angry customers

C. While all of the activities her have merit; the best preview is the one taht most closely mirrors the actual job. In this case, allowing the applicant to hear actual calls and see how a representative handle them provides the most realistic look at what the position entails.

Which of the following OSHA violations is the LEAST serious? A. repeat B. other-than-serious C. de minimis D. willful

C. Willful violations are the most serious, followed by serious, other-than-serious, and de minimis. De minimis violations have no direct or immediate relationship to job safety or health and carry no penalty.

In what succession plan approach are three or more potential candidates identified, all with various levels of readiness for promotion? a) Co-sourcing b) Labor market c) Replacement plans d) Talent pool

D

In which stage of project management are the plan deliverables and timelines most likely to be established? a) Implementation b) Evaluation c) Initiation d) Planning

D

The cost of living in the Silicon Valley of California has made it difficult for educational institutions to recruit teachers. This is because teachers cannot typically afford the housing costs in the areas where the schools are located. This is the best example of which of element of a SWOT audit? a) Strength b) Weakness c) Opportunity d) Threat

D

Leaving company devices and desk drawers unlocked is which of the following? A. an office security issue B. an asset security issue C. a building security issue D. a personnel security issue

The correct answer is A. Unlocked computer screens and desk drawers are examples of vulnerabilities in office security.

An organization that becomes entrenched in rules and policies and is resistant to change is in what phase of its organizational evolution? A. Maturity B. Growth C. Introduction D. Decline

D. An organization is in the decline phase when it becomes resistant to change and entrenched in the status quo.

What is organizational picketing?

This form of picketing is done to induce employees to accept the union as their representative

What is the first step in developing an RFP? A. conduct a needs assessment B. conduct internal benchmarking C. contact potential vendors D. set a deadline for submission

The correct answer is A. Developing the RFP should begin with a needs assessment, including the objectives and budget for the project, as well as whether it's a one-time project or an ongoing outsource relationship.

A narrow span of control is typically used when A. there is a shortage of managers B. a team effort is not required C. subordinates are experienced D. tasks are complex

D. "span of control" refers to the number of individuals who report to a supervisor. Narrower spans of control are typically used when tasks are complex, subordinates are poorly trained or inexperienced, or a team effort is required

Which of the following is the first step in the outsourcing process? A. Define the budget B. Outline the implementation process C. Create a request for proposal D. Conduct a needs analysis

D. A needs analysis is always the beginning point when outsourcing a product or service. All potential users of the product or service should have input into defining the goals, expectations, and scope of the product or service. Once this is done, a realistic budget can be established.

An employee who is on FMLA leave because of a serious health condition decides not to return to work at the end of 12 weeks. In this situation, the A. employer may not charge the employee for any health insurance premiums paid on behalf of the employee while the employee was on leave B. employee is entitled to maintain health benefit coverage for 18 months from the date the leave began C. employee may extend the leave for as long as necessary, with the approval of a physician D. employee is entitled to maintain health benefit coverage for 18 months from the date the leave expired

D. According to COBRA, the employee is entitled to maintain health insurance for 18 months, beginning at the expiration of the leave, which coincides with termination of employment

Which of the following statements about adult learners is true? A. they are more open to new ideas and alternative options than are student learners. B. they are not motivated by external incentives since self-satisfaction is most important C. they can assess their own progress and do not need feedback from the instructor D. they are inclined to refer to past experiences and want opportunities to share them

D. Adult learners are experience-based and have a wide variety of experiences to share. However, as a result, they may have formed habits and assumptions that make them less open to new ideas. While self-satisfaction is important, so are external rewards. Adult learners want to know how they are progressing and want feedback.

Which of the following BEST describes an organizational needs assessment? A. it uses employee interviews to determine training needs B. it identifies how well individuals perform their jobs C. it compares job requirements and employee knowledge and skills D. it identifies the KSAs employees will need in the future

D. An organizational analysis is concerned with the big picture. It identifies the skills, knowledge, and abilities employees will need in the future as the organization and their jobs change. A and B are examples of individual analysis, which focuses on individual employees and how well each performs his/her job. C is an example of task analysis, which starts with job descriptions and specifications that provide information on expected performance. Gaps between job requirements and performance indicate training needs.

Which of the following types of interviews focuses on how the applicant handled previous situations? A. stress B. directive C. structured D. behavioral

D. Behavioral interviews operate under the premise that past behavior predicts future performance. An interviewer focuses on how the applicant previously handled real situations and looks for three key elements--a description of the situation, the action taken, and the outcome

Diversity among team members will likely discourage which of the following? A. groupthink B. unionization C. creative conflict D. targeted recruiting

The correct answer is A. Diversity discourages groupthink and is more likely to broaden the scope of possible solutions by yielding different approaches to a problem.

Which of the following activities are both parties permitted to do during an election campaign? A. campaign in the polling place during election hours B. present speeches during working hours but before the polls open C. distribute literature in the polling area while voting occurs D. solicit individual votes outside the polling area while voting occurs

D. Campaigning by either party in or around the polling place during election hours is prohibited. However, literature may be distributed, and oral solicitation may occur outside the prescribed area until voting is over. Employers have a captive audience in their employees can present speeches during working hours, but they cannot do so within 24 hours of the election.

When an employer makes conditions for an employee so intolerable that the employee resigns, it is known as A. disparate treatment B. negligent hiring C. employment-at-will D. constructive discharge

D. Constructive discharge is considered an exception to the employment-at-will concept. Constructive discharge may fall into the category of retaliation and could play a part in civil rights cases or could involve legal issues in the areas of health and safety.

Which of the following is true about an EEOC investigation of an employee's charge of discrimination? A. the EEOC field offices will accept complaints filed within one year of the alleged discrimination B. the EEOC must initiate legal action if efforts at conciliation fail C. the EEOC can issue a right-to-sue letter that prevents further legal action D. the EEOC can ask an employer to participate in mediation before it begins an investigation

D. Depending on an initial priority assessment, the EEOC may request that both parties attempt mediation first. A is incorrect because claims must be filed within 180 or 300 days of the incident, depending on the jurisdiction. B is incorrect because although the EEOC can initiate legal action, it is not required to do so. C is incorrect because a right-to-sue letter gives the employee the right to file suit in court.

Which of the following is typically par of an environmental scan? A. Analysis of the organization's labor contract B. Analysis of minority employees within the organization C. Review of internal salary survey data D. Analysis of unemployment rates

D. Environmental scans focus on external changes and attempt to identify and interpret strategic issues that affect the company. D. is the only choice with an external focus. The other choices listed all deal with internal activities.

Which of the following statements about Medicare is true? A. Parts A and B of Medicare are mandatory B. Eligibility is dependent on a person's income and ability to pay C. Only retired individuals age 65 and over are eligible D. Medicare is secondary for employees over 65 covered by another health plan

D. For working employees over the age of 65, the employer's health plan is primary. A is incorrect because only Paet A of Medicare is mandatory. B is incorrect because eligibility is not dependent on income or ability to pay. C is incorrect because you don't have to be retired to be eligible, all individuals are eligible at age 65, whether or not they are still working.

Which of the following is a comparative appraisal method? A. checklist B. forced choice C. graphic scale D. forced distribution

D. Forced distribution is an appraisal method in which the appraiser compares the performance o each employee with that of others. Employees are rates and placed at different points along a bell-shaped curve. The method requires some employees to be placed in the lowest section and some in the highest, regardless of performance. The other options--checklist, forced choice and graphic scale--are all simple appraisal methods that require the appraisers to make an employee's performance on a designated form.

Which of the following is generally NOT a function of the payroll department? A. calculating Social Security and Medicare tax B. keeping a master file of employment records for the federal government C. deducting garnishments and tax levies D. contracting for pay surveys

D. HR professionals are responsible for developing a pay system and contracting with outside vendors for pay surveys. Payroll professionals are responsible for complying with federal, state and local payroll regulations, managing payroll reporting and record retention, and controlling the security of payroll information. Payroll and HR professionals work together to share and ensure that changes to employee records are reflected in both of their systems.

Which of the following has FIRST priority for inspection? A. catastrophes and fatal accidents B. employee complaints C. high-hazard industries D. imminent danger

D. Imminent danger has the highest priority; in such situation, there is a reasonable certainty that death or serious physical harm could occur immediately or before the situation could be corrected through normal enforcement procedures. Catastrophes and fatal accidents are defined as those taht require the hospitalization of three or more employees; they receive second party priority. Employee complaints regarding alleged violations of standards or unsafe working conditions receive third priority. High-hazard industries are assigned by OSHA to a high-hazard list on the basis of factors such as death, injury, illness, and employee exposure to toxic substances. They receive fourth priority.

Which of the following should be avoided in an offer letter? A. requiring a signature on a duplicate copy of an offer letter B. mailing employees informational brochures before they start work C. clarifying contingencies such as a medical exam D. quoting salary terms in an annual format

D. In an offer letter, state salary terms in hourly or monthly figures. If an employee is terminated, the annual figure could become binding. The other practices listed are sound employment practices and should not be part of the offer process

Which of the following involves a neutral third party resolving an impasse between an employer and a union? A. mediation B. common suites C. inside bargaining D. excelsior intervention

The correct answer is A. Mediation involves a neutral third party being presented information from both sides and making an informed decision on how to resolve the impasse.

A field HR unit that reports to the general manager of the unit and has "dotted line" accountability to corporate HR is referred to as what type of structure? A. Departmental B. Functional C. Matrix D. Divisional

D. In the divisional structure, divisions are separated on the basis of product, market, or region. All of the resources necessary to manufacture, sell, or supply the product are put under the control of the division. There is usually a "dotted line" accountability to corporate HR, but the field unit actually reports to the general manager of the division. The matrix relationship creates a dual reporting structure, and the functional structure has a single reporting relationship directly with a manager in a like functional area.

Which of the following determines the relative worth of each job in an organization? A. job analysis B. job specification C. job content D. job evaluation

D. Job evaluation determines the relative worth or each job by establishing a hierarchy of jobs within an organization. It is intertwined with a concern for internal pay equity.

A learning organization is an organization that: A. encourages each department to function independently B. advocates internal rather than external networking C. takes responsibility for the learning of its employees D. manages change and adapts to its environment

D. Learning organizations encourage constant networking both internally and externally. Change is embraced and managed, and learning experiences alter the organization's behavior. In such an organization, employees are responsible for their own learning.

Employers may require an employee on a WC leave to return to this assignment before they are able to resume their full duties. a. ADA b. Part-time c. Temporary d. Light duty

D. Light duty is restricted work activity. It must be ordered by the physician. It can be the same job with restricted duties or a different job.

Lockout/Tagout is the term used when: A. Employees strike and the employer locks them out B. An OSHA standard details how drugs must be kept safe and secure C. Personal Protective Equipment must be inspected for safety D. Machinery needing repairs must be made inoperative with locks

D. Lockout-the machinery needing repairs or adjustments must be made inoperative. Tagout-identifies the person(s) responsible for the lock

The Toxic Substance Control Act of 1976 requires that A. employees be required to wear protective equipment B. employers adopt sex-specific fetal protection policies C. physical conditions be evaluated by an outside consultant D. certain chemicals be pretested for safety

D. The Toxic Substance Control Act required the pretesting of certain chemicals to protect employees from chemicals, vapors, gases, and fumes that may pose significant health risks. B is incorrect because Title VII regulates fetal protection policies and forbids sex-specific protection policies that do not fall within BFOQ exception. C is incorrect because the law does not specify testing by an outside consultant.

Which of the following is a key responsibility of the operations function within an organization? A. ensuring that the product is priced profitably B. communicating the benefits of the product to customers C. deciding which features and benefits will appeal to customers D. determining the ability of the company to meet product demand

D. Operations departments are concerned with their ability to produce top-quality product that meets the demands of the marketplace. The operations department sets detailed time estimates that allow them to determine their capacity (ability to produce product). Based on forecasts, they develop a schedule that allows them to operate at capacity.

Which of the following quality tools shows how each item contribute to the total effect and allows users to focus on the frequency or impact of causes? A. run chart B. control chart C. histogram D. pareto chart

D. Pareto chart indicates the most frequent causes to the problem, although they may not be the most important in relation to the goals of our business and customers. A Pareto chart allows users to focus on the problems that offer the greatest potential for improvement. For example, you can determine which errors occur most frequently and concentrate on them.

Which of the following procedurally assists employers in complying with federal regulations against discrimination? A. Executive Order 11246 B. Congressional Accountability Act C. Title VII, Civil Rights Act D. Uniform Guidelines on Employee Selection Procedures

D. The Uniform Guidelines cover all aspects of the selection process including recruiting, testing, interviewing, and performance appraisals. Their purpose is to assist employers in complying with federal regulations against discrimination. The other choices set forth the law but do not assist employers in complying with it.

What is the purpose of workers' compensation legislation? a. To protect employers from unsafe working conditions b. To give workers paid time off when ill or injured c. To make employers liable for work related injuries d. To cover income and medical costs for work related injuries.

D. Pays for, full medical expenses, partial lost income, rehabilitative assistance, re-training (if necessary), monetary settlement, death benefit for dependents

The most likely circumstance in which a company will provide outplacement services for terminated employees. A. layoff B. bankruptcy C. hostile take over D. natural disaster

The correct answer is A. Outplacement services are most typically provided to employees who have been laid off.

Right-to-work laws allow states to A. establish emergency strike provisions when a strike threatens public welfare B. maintain a list of arbitrators to assist in contract settlement C. pay less than federal minimum wage D. forbid compulsory union membership

D. Right-to-work laws were established by some states, with the permission of the federal government, to forbid compulsory union membership.

Which of the following learning curves describes how most routine tasks are learned? A. plateau B. S-shaped C. increasing returns D. decreasing returns

D. Routine tasks are learned rapidly at first, and then the rate of improvement slows. Learning to use a computer mouse is an example.

An employee group made up of members from various areas in the organization concerned with safety issues is commonly called a: A. OSHA incentive B. safety relations forum C. work quality committee D. safety committee

D. Safety committees are responsible for making safety inspections and recommending changes. They are also promoting safety awareness.

Which of the following legislation encourages employers to hire targeted groups of job seekers? A. Walsh-Healey Act B. ADEA C. Service Contract Act D. Work Opportunity Tax Credit

D. The Work Opportunity Tax Credit encourages employers to hire people from nine targeted groups. As an incentive, the employers' federal tax liability decreases

Which of the following is usually considered a lawful practice? A. featherbedding B. slowdowns C. wildcat strikes D. sympathy strikes

D. Sympathy strikes are not prohibited by the Labor-Relations Act. A worker may refuse to cross a picket line at another employer's place of business when there is a strike approved by a majority union. In some cases, a union contract may have clauses that prohibit such a strike, but federal law does not prohibit such action. Featherbedding (requiring more workers than necessary), slowdown (partial or intermittent strikes), and wildcat strikes (strikes in violation of a no-strike provision are not lawful activities.

A mechanic is sent to an emissions school to learn the latest trouble-shooting techniques. The goal of the training is to prepare lead mechanics to train other shop mechanics. The training must be focused at what level in order to accomplish this goal? A. analysis B. application C. evaluation D. synthesis

D. Synthesis is the level at which a person can respond to new situations, trouble-shoot problems, and train others.

Which of the following is likely to be the result of continued advances in technology? A. training will be available only to those with high-speed internet access and voice-activated computers B. privacy issues will be eliminated by security advances such as iris and fingerprint scans C. In-house software will become increasingly desirable as application service providers become less flexible D. Electronic signatures will expand the types of transactions that can be done on the web

D. The Electronic Signature in Global and National Commerce Act mandates that an electronic signature (with few exceptions) carries the same weight as a pen-and-ink signature. This has opened the door to a variety of new types of actions that can be done online. Although training will be increasingly delivered online, it will not necessarily require the most sophisticated system in order to participate. Privacy issues will increase rather than decrease as companies try to determine what employees' rights are and whether sophisticated tracking and security procedures violate those rights. Finally, application service providers (ASPs) will be used more frequently for routine tasks such as time and attendance tracking and other administrative tasks that can be done more cost-effectively on a subscription basis.

What is integrative bargaining?

This type of bargaining takes place when there is more than one issue to be resolved

At a business supply company, account managers are responsible for collecting unpaid bills. The company decides to send account manager to a training course on bill collection. A successful account manager thinks that the course will be interesting but doubts that the company will give account managers time for collection activities. In this case, the training is likely to be minimally successful because the account manager A. seems unmotivated to learn new skills B. lacks the ability to learn new skills C. does not see how the training relates to the job D. doubts that the company will reinforce the learning

D. The account manager seems interested in learning, and, since he is a successful manager, we assume that he is capable of learning collection skills. Although the manager understands why such a skill is desirable, there is a perception that the company may not actually support the use of the skill. This uncertainty will lessen the training's effectiveness.

It is within the employer's rights to do which of the following? A. take pictures of employees going to and from union meetings B. question employees about union membership or activities C. provide preferential treatment of several unions trying to organize employees D. point out a union's strike history and the economic consequences of strikes

D. The employer provide facts about past strikes may discuss the general economic repercussions of strikes, provided the employer does not threaten a loss of jobs or wages. A is incorrect because an employer cannot engage in surveillance or intimidate employees who support a union. B is incorrect because it is unlawful for employers to question any employees about union activities. C is incorrect because the employer may not provide preferential treatment to a particular union. Since the employer is theoretically sitting on both sides of the table by supporting a union, the employer may be denying employees valid representation. A, B, and C constitute employer unfair labor practices.

The Financial Accounting Standards Board (FASB) is a private body that A. determines which compensation plans a company should offer to its employees B. interprets IRS revenue rulings if requested by an organization C. enforces accounting standards of for-profit and not-for-profit organizations D. determines how financial information should be reported to shareholders

D. The formation of the FASB marked the separation of the standards-setting process from the accounting profession. The FASB derives its authority form the Securites and Exchange Commission and decides how financial executives should report financial information to shareholders

Which of the following is NOT within the responsibility of the safety committee? A. conduct area safety inspections and evaluate hazards B. observe employees for compliance with protective measures C. review accident and injury data D. complete OSHA paperwork for accidents and illnesses

D. The safety committee has an ongoing responsibility to monitor safety data, review procedures and equipment, evaluate hazards, and investigate accidents. However, the committee is not responsible for the completion of OSHA paperwork.

What is arbitration?

Is the final step in the formal grievance process

Which of the following statements about Title VII of the Civil Rights Act is true? A. all employees must have equal working conditions B. discrimination against race and sexual orientation is prohibited C. employers must provide sexual harassment training for all employees D. employees must have an equal opportunity to participate in training

D. This law was passed to prohibit employment discrimination against protected classes. Several state laws have expanded these classes to include sexual orientation or public assistance status, but this is not a condition of the federal law. The law does not mandate equal working conditions or specific training programs but does require that all employees have equal opportunity to participate in training.

The president vetoes a bill and sends it to the 435-member House of Representatives. When the House attempts to override the veto, 360 representative are in attendance. How many representatives must vote for the bill? A. 180 B. 218 C. 235 D. 240

D. Two-thirds of the members who are present (in sufficient numbers for a quorum) must vote to override the veto.

In voluntary arbitration, both parties must A. agree to consider the judgment of the arbitrator B. waive their rights to appeal the award C. agree to a timeframe for settlement of the dispute D. accept the decision of the arbitrator as binding

D. Voluntary arbitration means that both parties willingly submit to arbitration and agree to accept the decision of the arbitrator as final and binding. They do not waive their appeal rights, but these appeal rights are very limited. The Supreme Court has stressed that any decision based on interpretation of the contract should be final and not questioned by the courts.

The PRIMARY reason employee handbooks should be carefully reviewed is because they A. provide new employees with an impression of the company B. may be the primary method of employee communication C. may be viewed by the company's competitors D. may create an enforceable contract

D. While all the answers may be true, the most important reason to review employee handbooks is to avoid inadvertently creating permanent, enforceable contractual obligations. Employees will expect to receive the benefits and follow the procedures outline in the handbook. Therefore, outdated benefits or procedures can create liability for the company.

Which of the following statements about defined contribution plans is true? A. employees have an ongoing responsibility to contribute to the plan B. employees of not-for-profit or public-sector organizations are not eligible C. employers have an obligation to make a minimum contribution to the plan D. employees assume risk in relation to how well the plan does

D. once an employer's contribution is met, the employer has no further responsibility to contribute to the plan and assumes no risk related to how well the plan does. Employees assume the risk, which is related to inflation and interest rates over the years. A is incorrect because employees may not to contribute to the plan. B is incorrect because not-for-profit and public-sector employees may participate in comparable plans such as 403b and 457 plans. C is incorrect because employers have an obligation to contribute a predetermined amount to the plan.

What is a comprehensive analysis of an organization's security vulnerability? A. vulnerability appraisal B. Homeland Security obligation C. OSHA audit D. security audit

D. security audit is also known as a vulnerability analysis.

Which of the following learning activities would appeal primarily to an auditory learner? A. diagrams and illustrated handouts B. group activities and model building C. lecture and review tapes D. video demonstrations and field trips

DC Auditory learners learn best by listening. They prefer to listen to lectures and talk things through. They often like to listen to audiotapes and will frequently record lectures. They also benefit from reading aloud.

What was the Labor-Mgt. Reporting and Disclosure Act or the Landrum-Griffin Act?

Established a "Bill of Rights" for union members

An injury only requiring ________ does not have to be reported to OSHA.

First Aid

What was the Labor-Management Relations Act or the Taft-Hartley Act?

Forbade employers to establish "sweetheart contracts", made closed shops illegal

Employers shall furnish . . . a place of employment which is free from recognized hazards that are causing or are likely to cause death or serious physical harm to his employees" is reflective of what OSHA clause?

OSHA General Duty Clause--the employer is responsible for providing safe work environment

What was the Railway Labor Act?

Passed to reduce labor conflict and the possibility of transportation strikes

What was the National Labor Relations Act or the Wagner Act?

Prohibited unfair labor practices and established the National Labor Relations Board

What is a wildcat strike?

Strikes that are not sanctioned or stimulated by the union

What is a tripartite panel?

Term for a three-member arbitration board

What is featherbedding?

Term that refers to unions requiring the employment of more workers than is necessary

Debra works for Access Communications and has revealed to authorities that the working conditions in the warehouse are unsafe. Which of the following explains why Debra won't be discharged by the company? A. She is reporting an unsafe work environment B. She is a union member C. She is an at-will employee D. She is an outplaced employee

The correct answer is A. A company cannot fire a whistleblower for reporting unsafe work environments.

Over the last 30 years union membership has declined sharply in _______________. A. the private sector B. the public sector C. both the public and private sectors D. neither the public nor private sectors

The correct answer is A. Over the last 30 years union membership has declined sharply in the private sector but has remained roughly constant in the public sector.

Which of the following is a provision of the Patient Protection and Affordable Care Act of 2010? A. Employers with more than 50 full-time employees are required to provide affordable coverage. B. All employees who have two years of service must be eligible to participate in an organization's pension plan. C. An employee who has worked for a covered employer for at least 12 months may apply for health benefit subsidies or receive tax credits. D. Individuals who are self-employed will be allowed to take up to 80 percent of their healthcare expenses as a deduction for tax purposes.

The correct answer is A. A key provision of the PPACA, commonly known as "Obamacare," is that employers with more than 50 full-time employees are required to provide affordable coverage to their employees.

What is the major disadvantage of functional departmentalization? A. It reduces communication between departments. B. It only works well in very large companies. C. It is more costly than product departmentalization. D. It is the least efficient way for small companies to organize themselves.

The correct answer is A. A system of functional departmentalization effectively reduces communication between departments, resulting in a silo effect that promotes parochialism and sub-optimizing.

What is the purpose of a utilization analysis? A. A utilization analysis shows how a company's workforce compares to the available labor supply in terms of gender, race, and ethnic composition B. A utilization analysis shows how a company's workforce compares to the surrounding population in terms of gender, race, and ethnic composition C. A utilization analysis shows how a company's workforce compares to the national averages in terms of gender, race and ethnic composition D. A utilization analysis shows how a company's workforce compares to its affirmative action goals in terms of gender, race, and ethnic composition

The correct answer is A. A utilization analysis shows how a company's workforce compares to the available labor supply in terms of gender, race, and ethnic composition.

Which of the following budget accounts is not a liability? A. Accounts Receivable B. Accounts Payable C. Accrued Expenses D. Mortgage Expense

The correct answer is A. Accounts receivable is money owed to the business by customers, and is classified as an asset. Accounts payable, accrued expenses, and mortgage expenses are all monies that the company will have to pay out at some time in the future, and are therefore classified as liabilities.

In which case did the Supreme Court rule that the employer must provide back pay for employees who suffered monetary loss due to racial discrimination? A. Albemarle Paper Co vs Moody (1975) B. McDonell Douglas Corp vs Green (1973) C. Griggs vs. Duke Power (1971) D. Automobile Workers vs Johnson Controls (1977)

The correct answer is A. Albemarle Paper Co. v. Moody established the principle that criteria for employment tests must be tied directly to job requirements, and must not be based solely on supervisor rankings. It also ruled that employers must provide back pay for employees who suffered monetary loss due to racial discrimination.

Which of the following is an advantage of external hiring? A. can help support diversity and achieve EEO goals B. provides greater motivation for good performance C. provides better opportunity to focus on and assess present employees' abilities D. allows the employee to make changes base on constituent groups' demands

The correct answer is A. An advantage of external hiring is that it provides greater diversity and can help the organization achieve its EEO goals.

An unauthorized work stoppage, while a valid contract is still in place. A. wildcat strike B. boycott C. lockout D. grievance

The correct answer is A. An unauthorized work stoppage, taken while a valid contract is still in place, is known as a wildcat strike.

Hasan is a Graphic Design Artist for a marketing firm. He recently submitted the third design that failed to impress his superiors for a new ad campaign. After this latest rejection, he meets with his Manager, Niko, who tells him that the team just doesn't like the designs. Hasan asks Niko for guidance on what the company is trying to convey with the ad campaign, but Niko only tells him not to do what he's already done. Which communications principle needs to be adjusted most in this situation? A. vision B. fairness C. honesty D. sensitivity

The correct answer is A. At the end of the discussion, both the manager and employee should have a clear, shared plan for what should happen in the future. Neither Hasan nor Niko seem to have a clear vision for the future in terms of what management wants for the ad campaign, which makes it hard to provide effective feedback. Both Hasan and Niko should have a clear understanding of what should be done differently.

What is the principal downside of using surveys to gather data about employee attitudes? A. employee surveys provide only limited information B. employee surveys can be very expensive C. employee surveys often yield unreliable data D. employee surveys cannot be used to track data

The correct answer is A. Because they contain a finite number of questions and employ forced-choice responses, surveys provide only limited information and cannot easily get at the fundamental attitudes and beliefs that underlie employee attitudes.

Which of the following is not an appropriate goal for a business strategy? A. achieving perfect competition B. achieving competitive advantage C. achieving imperfect competition D. achieving product differentiation

The correct answer is A. Business strategies generally seek to maximize company profits; Choices B, C, and D are all goals that would be consistent with profit maximization. Achieving perfect competition, by contrast, is a goal that would make the business environment more difficult and likely translate to lower profits over the long term.

Burr Firearms Corp. convened a formal meeting of its HR staff and a group of outside HR consultants to forecast workforce needs over the next two years. This is an example of ________. A. nominal group technique B. Delphi technique C. trend projection D. job bidding

The correct answer is A. Convening a panel of experts for an in-person, structured meeting to come to an agreement about workforce needs is an example of the nominal group technique.

If Freddie is a supervisor who focuses on short-range planning, which of the following is he most likely working on? A. creating a succession plan B. meeting broad organizational goals C. assessing strengths and weaknesses D. formulating the organization's philosophy

The correct answer is A. Creating a succession plan falls under projected staff requirements, which is part of an organization's short-range planning.

Which category of employers are not required to submit written emergency action plans and fire prevention plans to OSHA? A. employers with ten or fewer employees B. employers in low-risk industries C. employers with no history of OSHA violations D. employers who have passed an emergency readiness test administered by OSHA

The correct answer is A. Employers with ten or fewer employees are not required to submit written emergency action plans and fire prevention plans to OSHA, though such plans should still be developed.

If you are a private employer, your workers are eligible for leave under the Family and Medical Leave Act under what circumstances? A. If the employee has been employed for 12 months, and she works at a facility where the company employs at least 50 people within 75 miles of the facility B. If the employee has been employed for 24 months, and she works at a facility where the company employs at least 50 people within 75 miles of the facility C. If the employee has been employed for 12 months, and she works at a facility where the company employs at least 75 people within 50 miles of the facility D. If the employee has been employed for 24 months, and she works at a facility where the company employs at least 75 people within 50 miles of the facility

The correct answer is A. FMLA applies to private-sector employees who have worked for their employer for 12 months (and a minimum of 1,250 hours) and who work at a facility where the company employs at least 50 people within 75 miles of that facility. It is important to note that a single facility itself does not have to employ 50 people; if the company has multiple facilities within a 75-mile radius of each other, FMLA applies when total employment at those facilities equals or exceeds 50.

If Linda, an HR Manager, wants to institute a wellness program that focuses on educating employees on the benefits of fitness, with which of the following is she most likely concerned? A. employee stress B. substance abuse issues C. workplace violence D. repetitive motion issues

The correct answer is A. Fitness programs can help employees combat stress, which is known to cause erratic eating habits, high blood pressure, elevated cholesterol levels, and depression.

Why might some companies decide to provide employees with a flexible benefits package? A. because different employees at different times in their lives have varying needs and wants B. because flexible benefits packages are cheaper to administer C. because employees tend to want the same types of benefits throughout their careers D. because flexible benefits packages reduce healthcare costs

The correct answer is A. Flexible benefits packages let employees choose which benefits they want, because different employees have varying needs at different points in their lives.

Which of the following are effective strategies for getting employees to embrace change? A. Foster participation and reward employees who cooperate B. Tightly control the flow of information and micromanage the change process C. Assume that employees will resist the change and terminate anyone who does not cooperate D. Start by working with line managers and get the approval of top management later in the process

The correct answer is A. Fostering participation and rewarding employees who cooperate are effective change management strategies.

Which of the following statements describes how HR best contributes to the success of a firm's business strategy? A. Sound HR practices increase the firm's organizational capabilities, creating a competitive advantage. B. Sound HR practices enable a firm to minimize payroll, creating a cost advantage. C. Sound HR practices enable a firm to introduce automation at a faster rate, creating a cost advantage. D. Sound HR practices improve the firm's outbound logistics, creating a competitive advantage.

The correct answer is A. HR's principal goal should be to increase the firm's organizational capabilities; superior workers are an enduring source of competitive advantage because workers cannot be as easily replaced or imitated as products or technology can be. Firms that seek to minimize payroll as a primary goal will find themselves in a "race to the bottom" against factories in low-wage regions or nations. HR does not necessarily have a direct impact either on automation or outbound logistics.

A company's sales department has struggled to retain new employees. Employees who have gone through the company's training program tend to praise the program itself, but criticize its leadership. The Director of HR decides to conduct an experiment to assess the performance of the head trainer. She asks two recently hired salespeople to go through the training with the head trainer, whom she labels as the independent variable. What is the flaw in the design of this experiment? A. It did not establish a control group B. It did not establish an experimental group C. The salespeople should be the independent variable and the head trainer the dependent variable D. The head trainer should be the dependent variable and the program itself the independent variable.

The correct answer is A. If the company labels the HR leader the independent variable, then one of the new sales hires should partake in training with the HR leader, and the other should receive training from someone else. As designed, the experiment has no control group.

In a value-chain analysis, HR management is most closely related to which of the following firm activities? A. procurement B. inbound logistics C. operations D. outbound logistics

The correct answer is A. In a value-chain analysis, HR management and procurement are both considered to be support activities. Inbound logistics, outbound logistics, and operations are all considered to be primary activities.

What is the principal constraint in refining your company's computer model to produce the most accurate HR forecasts possible? A. cost B. time C. availability of data D. privacy risks

The correct answer is A. In seeking to improve computer models for probabilistic forecasting, HR must balance the benefit of greater accuracy against the increased cost of introducing multiple new variables into the simulation.

Kazim was recently hired at an investment firm. His new manager, Donna, is very controlling and closely supervises all of her employees. She explains to Kazim that her reasoning for doing so is that she believes people need money to survive; they aren't coming here because they want to, but rather because they have to. Which approach to management best describes Donna? A. Theory X B. Theory Y C. Transformational D. Classical conditioning

The correct answer is A. Managers who work according to Theory X view their employees as essentially uninterested in work. Donna seems to believe that employees want to do as little as possible and are not interested in taking responsibility for their work.

After multiple warnings and meetings, Jerome decides that he will fire Stephanie, a Graphic Designer who regularly arrives to work late and has a consistently poor performance record. However, Jerome is concerned about how others will view this dismissal, as Stephanie is deaf. Which of the following best explains why Jerome should be very thorough and precise when he explains his decision? A. Stephanie is a member of a protected class B. Stephanie might claim constructive discharge C. Jerome doesn't want this to be viewed as a layoff D. Jerome must ensure that Stephanie has access to outplacement services

The correct answer is A. People with disabilities are a protected class, protected by federal law from discrimination. Jerome should ensure that the dismissal is not seen as discriminatory.

As HR Director of Fisk Industries, you are conducting an employee satisfaction survey aimed at finding out how employees feel about salary, benefits packages, and vacation policy. This survey is a measure of ________. A. company culture B. company values C. company culture D. company mission

The correct answer is A. Quantitative opinion surveys, which measure how employees feel about their own job situations and the organizations, are climate surveys. By contrast, an effort to probe company culture, which reflects deep-rooted beliefs and long-standing rules of behavior, would most likely require extensive qualitative research, such as focus groups.

Which of the following is an example of a bad question to ask a focus group? A. Do you like the organization's new flexible hours policy? B. What is the most positive aspect of the new benefits plan? C. How do you think the merger will affect your department? D. What are the two most important changes in the company's new healthcare plan?

The correct answer is A. Questions that can be answered with a simple yes or no should be avoided.

________ is the process of choosing from a pool of candidates for a job with an eye toward hiring the most qualified applicant; ________ is the process of seeking and attracting candidates for given vacancies in an organization. A. selection; recruitment B. recruitment; selection C. operational planning; selection D. strategic planning; operational planning

The correct answer is A. Recruitment is the process of seeking and attracting candidates for given vacancies in an organization, while selection is the process of choosing from the pool of candidates for a job with an eye toward hiring the most qualified applicant.

Which of the following is an example of a benefit? A. sick leave B. cash bonuses C. hourly wage D. stock options

The correct answer is A. Sick leave is an example of an employee benefit.

The principal difference between a task force and a committee. A. A task force is formed to address a specific problem; a committee is formed to address ongoing issues B. A task force consists only of managers; a committee includes both managers and line employees C. A task force includes both managers and line employees; a committee consists of only managers D. A task force has fewer members than a comittee

The correct answer is A. Task forces address a single problem and are then disbanded; committees address ongoing issues and often have an open-ended lifespan.

Established the minimum wage. A. Fair Labor Standards Act B. Norris-LaGuardia Act C. Equal Pay Act D. Labor Management Relations Act

The correct answer is A. The Fair Labor Standards Act established minimum wage, maximum weekly hours, and overtime pay requirements in industries engaged in interstate commerce.

According to federal regulations, how long must employers retain records of consumer credit reports? A. They must be disposed of immediately. B. one year C. three years D. five years

The correct answer is A. The Fair and Accurate Credit Transactions (FACT) Act mandates that records of consumer credit reports must be disposed of right away.

Which of the following is an accurate statement of the Pareto Principle? A. 80% of the problems in an organization result from 20% of the possible causes B. 80% of the payroll in an organization goes to 20% of the employees C. in any organization, people rise to their own level of incompetence D. if something can go wrong, it will

The correct answer is A. The Pareto Principle holds that 80 percent of the problems in an organization result from 20 percent of the possible causes.

A conflict of interest is a violation of which fiduciary duty? A. duty of loyalty B. duty of obedience C. duty of good faith C. duty of care

The correct answer is A. The duty of loyalty states that a board member must the interest of the organizations of the organization first by avoiding conflicts of interest and not abusing his or her position for personal gain.

Which of the following is not a factor in a PEST analysis? A. production factors B. economic factors C. social factors D. technological factors

The correct answer is A. The four factors in a PEST analysis are Political, Economic, Social, and Technological.

Which of the following perspectives is not part of the balanced scorecard? A. Productivity Perspective B. Financial Perspective C. Customer Perspective D. International Business Process Perspective

The correct answer is A. The fourth perspective in the balanced scorecard is the Learning and Growth Perspective, not the Productivity Perspective.

What is a primary security risk for buildings located in highly trafficked areas? A. Employees might not recognize someone who shouldn't be there B. Thieves are more likely to target the building for break-ins C. Heavy traffic may keep law enforcement personnel from accessing the building D. Heavy traffic may impede employees' escape from the building in the event of an emergency

The correct answer is A. The presence of heavy traffic around the building may allow intruders to blend in with the crowd, and prevent employees from recognizing them due to the regular presence of unfamiliar faces.

In what phase of the instructional design process is the target audience identified and the content developed? A. design B. development C. implementation D. evaluation

The correct answer is A. The target audience identified and the content is developed during the Design phase.

Ayanna works for Guardian Inc., a cybersecurity firm. She recently took several courses on incident response and forensic analysis, for which she received a three percent raise. Which of the following pay systems has most likely been utilized? A. skill pay B. merit pay C. commission D. braodbanding

The correct answer is A. This is an example of skill pay, in which employees are rewarded for acquiring new skills.

Andrew and Maria are part of an interview team tasked with finding a new Chief Financial Officer after the current one announced her intention to retire. Andrew and Maria decide they will each create their own list of questions to ask candidates. They also decide that Andrew will ask most of the initial questions, and Maria will ask follow-up questions. Which type of interview will Andrew and Maria most likely conduct? A. panel interview B. directive interview C. patterned interview D. hypothetical interview

The correct answer is A. When several interviewers observe a candidate at once, this is called a panel or board interview. These may involve questions being asked by all interviewers on the panel, or there may be a particular interviewer who takes the lead and does most of the question-asking. In some cases, it is best to have multiple interviewers when trying to fill executive-level positions such as CFO.

What is the difference between a code of ethics and a code of conduct? A. A code of ethics describes the ideal standards the company tries to uphold in its business practices; a code of conduct describes the behavior it expects from employees. B. A code of ethics describes the behavior the company expects from its employees; a code of conduct describes the ideal standards that it tries to uphold in its business practices. C. A code of ethics applies to the board and senior management; a code of conduct applies to rank-and-file employees. D. There is no difference; the two terms are synonymous.

The correct answer is A. While the two terms are sometimes used interchangeably, they serve different purposes. A code of ethics describes the way the company seeks to behave in its business dealings; a code of conduct spells out the types of employee behavior that are considered acceptable and unacceptable.

Which of the following describes a wildcat strike? A. a strike initiated by workers against the instructions of their union B. a strike during which picketing workers engage in violent or destructive behavior C. a strike in which employees of one organization stop work in solidarity with striking workers of another unaffiliated organization D. a strike during which union negotiations reach an impasse

The correct answer is A. Wildcat strikes are strikes initiated by workers without authorization from their union, while a binding labor agreement is still in place. This makes such strikes illegal.

Some agricultural field supervisors are able to supervise as many as 60 to 75 field hands. This fact is consistent with which of the following statements? A. Agricultural work tends to be repetitive and stable, which allows for a larger span of control. B. Field hands tend to be highly educated and trained, which allows for a larger span of control. C. Agricultural work tends to be repetitive and stable, which calls for a smaller span of control. D. Field hands tend not to be highly educated and trained, which allows for a smaller span of control.

The correct answer is A. Work that is repetitive and stable, such as farm work, requires less supervision than more variable and complex work. As a result, the span of control for a field supervisor tends to be larger than for supervisors in non-agricultural work.

According to Keisha O/Marde, which of the following is a key metric in assessing an organization's retention rate? A. years of service B. industry benchmarks C. state of the career life cycle D. satisfaction with the onboarding process

The correct answer is A. Years of service is a key metric in assessing an organization's retention rate. An organization should generally be more concerned when someone leaves after fifteen years of service versus someone who leaves after two years.

Which of the following describes how business should be conducted within an organization? A. Mission statement B. Corporate values statement C. Income statement D. Vision statement

The correct answer is B. A corporate values statement describes how business should be conducted within an organization.

A company wants to forecast the net impact of a program on its bottom line. It weighs the expected expenses and advantages against each other. Unlike ROI, which generally looks only at the hard costs this __________ also looks at the soft costs. A. replacement cost B. cost-benefit analysis C. outlay cost D. human resource ROI

The correct answer is B. A cost-benefit analysis identifies all the costs associated with a particular program or action and weighs them against all the benefits that will result, to forecast the net impact on the company's bottom line. Unlike ROI, which generally looks only at hard costs, a CBA looks at soft costs, as well.

A framework for distributing both monetary and nonmonetary compensation is known as____. A. payroll B. total rewards strategy C. flexible benefits package D. competitive recruiting advantage

The correct answer is B. A framework for distributing both monetary and nonmonetary compensation is known as a total rewards strategy.

Luisa, a Manager at Drlcom Mobile, has 12 direct reports. What is her span of control? A. 6 B. 12 C. 24 D. It depends

The correct answer is B. A manager's span of control is defined as the number of workers who report to her.

A profit-sharing program is an example of what type of compensation? A. Sales bonus option B. Group incentive C. Deferred compensation D. Gainsharing strategy

The correct answer is B. A profit-sharing program is an example of a group incentive - an incentive that rewards individuals for their collective accomplishments. Gainsharing is also a type of group incentive, though it is based on gains in productivity rather than profits. Deferred compensation is a strategy that relates to retirement plans, while a sales bonus option is a form of commission.

What is the difference between a reference check and a background check? A. a reference check is more detailed than background check B. a reference check involves people who are familiar with the applicant's work performance, while a background check involves non-employment sources C. a background check involves people who are familiar with the applicant's work performance, while a refence check involves non-employment sources D. there is no difference; the two terms are interchangeable

The correct answer is B. A reference check involves people who are familiar with the applicant's work performance, while a background check involves non-employment sources.

Which of the following methods of employee involvement allows employees to make recommendations anonymously? A. committees B. suggestion boxes C. work teams D. task forces

The correct answer is B. A suggestion box allows employees to anonymously submit recommendations for the organization. This can be helpful for employees who may be anxious about speaking up personally.

What is the difference between a task force and a committee? A. Task forces are larger than committees. B. Task forces disband after the issue they were formed to address has been resolved, while committees are permanent. C. Committees focus mainly on analyzing organizational weaknesses to expose issues, which are directly addressed by task forces. D. Committees disband after the issue they were formed to address has been resolved, while task forces are permanent.

The correct answer is B. A task force is formed to address a specific problem and disbands once that problem has been solved. Committees address ongoing issues within an organization.

The sales department at Megacorp Consolidated forecasts that sales will increase by 18 percent in the coming year. The HR department, which knows that rising sales have historically been linked to increased hiring, develops a budget that assumes a commensurate increase in payroll costs for the following year. This is an example of ________. A. unit demand B. trend projection C. workload analysis D. ratio analysis

The correct answer is B. A trend projection is a prediction of future employment needs, based on an understanding of how other factors (e.g., sales, production, etc.) affect employment. In this case, HR understands that there is a close relationship between rising sales and increased employment; once the sales department projects a significant increase in sales, HR begins planning to increase employment to accommodate this trend

Which of the following is not one of Michael Porter's Three Grand Strategies? A. cost leadership B. innovation C. differentiation D. focus

The correct answer is B. According to Michael Porter, the Three Grand Strategies are Cost Leadership, Differentiation, and Focus. Innovation is an essential element of Differentiation, but it is not one of Porter's Grand Strategies.

Tim is a Sales Manager; he begins his weekly team meeting by announcing that the sales rep who secures the most letters of agreement by the end of winter will get three extra paid vacation days. While the younger team members look excited, the older team members look at each other and shrug. Which theory best describes this reaction to Tim's plan? A. McGregor's X or Y Theory B. Vroom's Expectancy Theory C. Maslow's Hierarchy of Needs D. Herzberg's Two Factor Theory

The correct answer is B. According to Vroom, after assessing a situation to determine whether an employee is capable of handling it, they will then determine whether the anticipated reward justifies taking on the task. While some of the younger members of the team may find more vacation time very appealing, older team members may not find the anticipated reward worth the hustle.

Which of the following is an advantage of promoting from within? A. creates a homogeneous workforce B. does not change the organizational hierarchy much

The correct answer is B. An advantage of internal promotion is that it provides greater motivation for good performance. Current employees will strive for the promotion when they know it is obtainable.

Which of the following scenarios best indicates that an employee may need coaching? A. an employee who drinks a lot of coffee in the morning B. an employee who struggles in handling new responsibilities C. an employee who demonstrates potential to become an executive D. an employee who likes to keep to themselves and avoids casual conversations

The correct answer is B. An employee who struggles in handling new or different responsibilities would benefit from coaching.

In which career phase does performance typically improve? A. exploration B. establishment C. maintenance D. disengagement

The correct answer is B. An employee's performance typically improves during the Establishment phase.

What part of a job description describes the chain of command in which the worker will operate? A. department B. reporting relationship C. responsibilities D. term of employment

The correct answer is B. An employee's reporting relationship -- to whom she reports, and who reports to her -- describes the chain of command in which she will operate.

FSA funds can be used for which of the following: A. travel expenses B. eyeglasses C. monthly rent or mortgage payment

The correct answer is B. As a benefit, a company may offer health flexible spending accounts (FSAs). With an FSA, an employee puts money aside, tax-free, to cover all but catastrophic health costs. FSA funds can be used for qualified medical expenses not covered by health insurance, including items such as eyeglasses and hearing aids.

The CEO announces that the company is moving to a new building in the next town over. The announcement frustrates Aubrey, a member of the IT staff, because the new location will add 30 minutes each way to his already long commute. Visibly distressed, Aubrey confronts Evelyn, his manager, and harangues her about his displeasure. Evelyn takes a moment, then asks if telecommuting on Tuesdays and Thursdays would make Aubrey's situation easier. Aubrey says yes, agrees to this new work schedule, and thanks Evelyn. Which of the following components of emotional intelligence should Aubrey work on for the future? A. self-awareness B. self-management C. social awareness D. relationship management

The correct answer is B. Aubrey is understandably upset with the move. However, change is inevitable for any organization, and it would benefit Aubrey to practice self-control of his emotions when confronting change at work.

Ramone just finished the final round of interviews and has narrowed the list of applicants down to two people. He decides his next move is to find out if either applicant has a criminal record. Which of the following will Ramone conduct to find this information? A. reference check B. background check C. screening interview D. past performance check

The correct answer is B. Background checks involve verifying that the applicant does not have a criminal record and has not falsified any records.

What is the major advantage of functional departmentalization? A. It is the most efficient way for small companies to organize themselves B. It promotes greater skill specialization C. It promotes greater cooperation between parts of the company D. It eliminates the duplication of equipment and effort

The correct answer is B. By allowing people who face similar problems and opportunities to work together, a system of functional departmentalization promotes skills specialization, allows for a highly efficient use of resources, and encourages the use of specialists and specialized equipment.

Gerry's Jeans compensates its sales employees through a combination of a base salary plus three percent of the employee's sales for the year. Which type of payment system does Gerry's Jeans utilize? A. merit pay B. commission C. profit sharing D. group incentive pay

The correct answer is B. Gerry's Jeans pays its employees through a commission system.

Which of the following is not one of the functions of an organization's mission and vision statements? A. to communicate the purpose of the organization to stakeholders B. to lay out the core principles by which the organization seeks to operate C. to guide development of organizational strategy D. to develop the goals that will gauge the organization's success

The correct answer is B. Choices A, C, and D are all functions of mission and vision statements, but Choice B describes the function of an organization's values statement.

Concerned about low retention, the CEO of a social media marketing firm asks employees what would make them happier; the employees overwhelmingly respond that they want more flexibility in their schedules. The CEO asks Sheila, the firm's HR Manager, to attend four bi-weekly sessions in which she and other HR managers listen to presentations about the benefits and challenges of hiring and managing remote employees. The attendees also have the opportunity to ask questions and discuss amongst themselves what they would do in certain situations. Which type of training session is Sheila most likely attending? A. virtual B. classroom C. simulation D. industry event

The correct answer is B. Classroom training is usually lecture-based and allows for participants to explore complex ideas. One drawback of classroom training is that it provides little to no hands-on experience.

Philipp is having some difficulty interacting with a coworker. When can Phillip's manager use coaching to improve the situation? A. only during Phillip's annual performance review B. at any appropriate time C. only during Phillip's lunch hour D. only at the end of the workday

The correct answer is B. Coaching is effective because it provides immediate feedback and is typically done in a casual manner.

Joaquin is a talented employee whose performance has slipped over the past month. His supervisor, Ida, decides to specialize Joaquin's skills. If she's confident Joaquin will develop these new skills quickly, how should Ida help Joaquin? A. by mentoring him B. by coaching him C. by counseling him D. by consulting with him

The correct answer is B. Coaching is focused on the short-term development of skills and improvement of performance. While this coaching can be provided by the employee's direct supervisor, Ida could also hire an external consultant to approach this issue from a new perspective.

Which of the following has most likely occurred if we believe Ronnie, a Systems Analyst who has accused his former manager, Linda, of constructive discharge? A. Ronnie was fired because he is African American. B. Linda has repeatedly harassed Ronnie C. Ronnie has yet to receive HR assistance in finding another job. D. Linda issued a suspension before a verbal or written warning for insubordination

The correct answer is B. Constructive discharge occurs when an employer has made working conditions so intolerable, through harassment or other means, that an employee feels forced to quit.

The Fair and Accurate Credit Transactions (FACT) Act mandates that consumer credit reports must be properly disposed of how long after an organization no longer needs them? A. one year B. immediately C. three years if stored electronically D. five years or until the final deposition of the lawsuit is tiled

The correct answer is B. Consumer credit reports must be properly disposed of immediately once the organization no longer needs them.

Which is of the following is NOT a legally required benefit? A. workers' compensation B. dental coverage C. unemployment compensation D. medicare

The correct answer is B. Dental insurance is a benefit provided at the employer's discretion; it is not a legally required benefit.

During what phase of the ADDIE process do the training designers identify the target audience? A. Analysis B. Design C. Development D. Evaluation

The correct answer is B. During the Design phase, the designers identify the target audience, develop training objectives, and develop content.

Due to the potential time savings of a new software, an organization decides to switch to it and utilizes Lewin's change process theory to do so. However, the organization becomes stuck in the unfreezing stage. Which of the following best explains why the organization cannot progress to the next stage? A. Employees are still getting used to the new software B. Employees are skeptical that the change is necessary C. Employees aren't actually saving time by using the new software D. Employees aren't actively trying to learn how to use the new software

The correct answer is B. During the unfreezing stage, leadership must convince members of the organization that change is essential and must begin immediately

An agency shop clause does which of the following? A. prohibit non-union employees from working at a union organization B. require all employees of a union organization to pay union dues, whether they are union members or not C. prevent the reopening of negotiations once a contract is in place D. prohibit both employees and management from initiating a work stoppage, such as a strike or lockout

The correct answer is B. Employees of an organization operating with an agency shop clause (an agency shop) must pay union dues even if they are not part of the union. Such clauses are intended to encourage all employees to join the union.

The process of forecasting possible risks to an organization and taking steps to mitigate their impact on operations is known as ________. A. compliance B. ERM C. an HR audit D. continuity planning

The correct answer is B. Enterprise risk management, or ERM, is the process of forecasting possible risks to an organization and taking steps to mitigate their impact on operations.

Firing an employee for specific, usually documented reason. A. termination at-will B. termination for cause C. justifiable termination D. statutory termination

The correct answer is B. Firing an employee for a specific reason is known as termination for cause.

Due to an ongoing recession, ABC Company must reduce its workforce. The CEO is confident, however, that the organization will be in a better financial position in six months, at which time they will be able to support a full workforce again. They decide the best approach is to reduce the number of hours of 20 employees by 25 percent. If the company is confident that they will be able to compensate a full workforce in six months, which of the following alternative practices will they execute in the meantime? A. attrition B. furlough C. demotions D. rolling layoffs

The correct answer is B. Furloughs amount to part-time employment, either by reducing the number of hours or the number of days worked. They can occur when an organization believes that an economic slowdown is temporary and they don't want to lose their workforce.

Which HR metric expresses how well workers in the organization are generating revenues, given the amount they are paid in compensation and benefits? A. Return on Investment B. Human Capital ROI C. Innovation Rate D. Cost Per Hire

The correct answer is B. Human Capital ROI measures the extent to which employees are generating revenues for the organization, in return for the investment that the organization has made in their compensation and benefits.

Under Sarbanes Oxley, attorneys are required to report breaches of fiduciary responsibility or violations of securities law either to the company's CEO or to its chief legal officer. If this does not result in a resolution, what is the next step attorneys must take? A. report the alleged violations to the SEC B. report the alleged violations to the board of directors C. report the alleged violations to OSHA D. report the alleged violations to the FBI

The correct answer is B. If the CEO or chief legal officer does not resolve the allegations, attorneys are required to report the alleged violation to the audit committee of the company's board of directors.

If an employer requires that employees are expected to contribute a fixed amount of money to their health coverage costs each year, which of the following coverage plans does that employer offer? A. flexible spending coverage B. defined contribution coverage C. variable contribution coverage D. preferred provider organization coverage

The correct answer is B. In defined contribution coverage plans, employees are expected to contribute a fixed amount to their health coverage costs each year.

Which of the following members of a business would not have a fiduciary duty? A. member of the board of directors B. silent partner C. CEO D. managing partner

The correct answer is B. In general, fiduciary duties or responsibilities apply to those who have a role in managing the business, either on a day-to-day basis or through a governance role. Thus, a member of the board would have fiduciary duties, as would the CEO or a managing partner. But a silent partner—that is, one whose role is limited to providing capital to the business, with no management responsibilities—would not have a fiduciary duty.

Based on the Miles-Snow typology of business strategies, which of the following strategic types is characterized by effort to retain current customers without having to innovate or grow? A. Prospector B. Defender C. Analyzer D. Reactor

The correct answer is B. In the topology of business strategies devised by Raymond Miles and Charles Snow, the Defender strategy focuses on stability; it seeks to retain the current customer base and improve internal efficiency, with little thought for innovation. The Defender strategy is essentially the opposite of the Prospector strategy.

Ramzy is in charge of hiring a new Senior Sales Analyst. He thinks two employees, Tonya and Justin, might already have the experience necessary for the position. He sends an email to everyone in the company outlining the available position and the requirements. Which of the following has most likely occurred? A. nepotism B. internal recruitment C. external recruitment D. employee requisition

The correct answer is B. Internal recruitment occurs when existing employees have the necessary skills and experience to fill the new job position. Ramzy is utilizing internal recruitment to see if an existing employee could fill the new job position.

Henry is about to conduct a semi-structured interview with a job candidate. Which of the following questions would be inappropriate for Henry to ask the candidate? A. Will you be able to work evenings if required to do so? B. Which, if any, religious holidays do you observe throughout the year? C. Can you provide documents that prove you are authorized to work in the United States? D. Will you be able to perform the necessary job functions if we accommodate your deafness?

The correct answer is B. Interviewers cannot ask candidates if they have religious affiliations, nor can they ask candidates whether or not they will work Sundays.

Mika is in charge of finding a Graphic Designer and has interviewed two exceptional candidates. However, both have time-consuming family commitments and have asked separately if they can work part-time. If Mika was already approved to create an alternative staffing arrangement, which type will she most likely choose? A. offshoring B. job sharing C. outsourcing D. job rotation

The correct answer is B. Job sharing appears to be the best fit for these two candidates. Both will have a responsibility for the success of the total job and share the duties of one full-time position.

What is a major difference between development and leadership development? A. leadership development is primarily concerned with the financial side of an organization, while management development is primarily concerned with the operational side B. leadership development aims to provide a comprehensive understanding of the entire organization, while management development is more narrowly focused on managerial skills and knowledge C. leadership development works with candidates already in the organization, while management development looks for candidates outside the organization D. there is no difference between the two

The correct answer is B. Leadership development takes a more comprehensive approach, while management development is more narrowly focused.

Which of the following can protect a company against a privacy misconduct lawsuit? A. the Stored Communications Act B. EPLI C. Professional Liability Insurance D. Errors and Omissions Insurance

The correct answer is B. Many companies purchase employment practices liability insurance (EPLI), which defrays some of the legal costs associated with defending a civil lawsuit brought by an employee. The Stored Communications Act is a federal statute that protects the storage of emails and other electronic communications. Professional Liability Insurance, also known as Errors and Omissions Insurance, protects a business against claims of negligence or inadequate work.

In which case did the Supreme Court rule that complainants face the initial burden of proof when bringing charges of discrimination against an employer? A. Albemarle Paper Co. v. Moody (1975) B. McDonnell Douglas Corp. v. Green (1973) C. Griggs v. Duke Power (1971) D. Automobile Workers v. Johnson Controls (1977)

The correct answer is B. McDonnell Douglas Corp. v. Green established that complainants must establish that they are members of a protected class, that he or she was qualified for the job to which he or she was applying (in the case of job applications), that he or she was rejected despite those qualifications, and that the position remained open for new applications after that rejection.

Outlawed the closed shop A. Wagner Act B. Taft-Hartley Act C. Civil Rights Act D. Equal Pay Act

The correct answer is B. The Taft-Hartley Act outlawed the closed shop.

Which of the following plans provides the least amount of choice in selecting a healthcare provider, but is best at controlling costs? A. point of service B. group health cooperatives C. preferred provider organization D. health maintenance organization

The correct answer is B. Members of a group health cooperative pay a fixed monthly fee for access to a specific collection of physicians, hospitals, and clinics. Out-of-network care is available, but at a higher price.

What is the main difference between outsourcing and offshoring? A. Outsourcing involves the use of foreign workers; offshoring does not necessarily involve foreign labor B. Offshoring involves the use of foreign workers; outsourcing does not necessarily involve foreign labor C. Offshoring is illegal; outsourcing is not D. Offshoring is more expensive than outsourcing

The correct answer is B. Offshoring involves shifting work from higher-paid American labor to less expensive foreign labor; outsourcing does not necessarily involve foreign labor.

Which of the following is a legal justification that employers can use to defend themselves against age discrimination charges? A. the dismissal was a financial necessity B. the dismissal was not related to the employee's age C. the dismissal was due to the employee's inability to keep up with others D. the dismissal was consistent with a policy the employer implemented in the previous year

The correct answer is B. One legal justification is that the dismissal had nothing to do with the employee's age, but rather something else such as layoffs or the closing of a department.

The process of contracting an internal business function to an outside organization that specializes in that specific function is known as ________. A. offshoring B. outsourcing C. privatization D. divestiture

The correct answer is B. Outsourcing is the process of contracting an internal business function, such as payroll or janitorial services, to an outside organization that specializes in that specific function.

Which of the following is NOT a form of performance pay? A. skill pay B. overtime C. merit pay D. commissions

The correct answer is B. Overtime is not a form of performance pay.

Reassigning duties and revising the job description are steps in what process? A. job analysis B. job redesign C. job evaluation D. job design

The correct answer is B. Reassigning duties and revising the job description are steps in job redesign.

Providing ergonomic desks, chairs, and computer equipment to employees is an effective way of protecting them from ________ hazards. A. lower back injury B. repetitive motion C. stress D. substance abuse

The correct answer is B. Repetitive motion hazards affect a worker's tendons and can lead to conditions like carpal tunnel syndrome. Ergonomic desks, chairs, and computer equipment can help mitigate these risks, as can fitness programs and training in the prevention of repetitive motion problems.

Which of the following performance appraisal methods would be effective in reducing the effect of biases from a manager who tends to be too harsh or too lenient in their performance appraisals? A. critical incident review B. forced distribution C. essay review D. field review

The correct answer is B. Requiring a manager to evaluate employees using a forced distribution system should minimize the effects of their biases by requiring them to place a certain number of employees in both the exemplary and unsatisfactory categories, based on their performance relative to their peers.

Scoping would take place during which stage of the RFP process? A. evaluation B. needs assessment C. vendor selection D. contract negotiation

The correct answer is B. Scoping is the process by which the organization that is preparing an RFP meets informally with potential vendors, to get a better sense of what the project will entail and how best to describe the desired products and services. This process is part of the needs assessment.

Skill variety, task identity, and autonomy are types of A. worker sensibility B. worker motivation C. worker characteristics D. job descriptions

The correct answer is B. Skill variety, task identity, and autonomy are three of the five varieties of worker motivation identified by Hackman and Oldham.

Changing an organization's mission statement is an example of which of the following? A. human process intervention B. strategic intervention C. management by objectives D. techno-structural intervention

The correct answer is B. Strategic interventions are designed to implement changes to the organization's vision or mission, as part of a strategic planning process. Change management is a type of strategic intervention.

Moe, an HR analyst at Canberra Foods, has constructed a model, based on eight years of internal data, which includes employee satisfaction surveys and exit interviews, to predict which Canberra employees are most likely to quit their jobs over the next two years. Moe's model is an example of ________. A. internal benchmarking B. strategic analytics C. external benchmarking D. prescriptive analytics

The correct answer is B. Strategic or predictive analytics involves the collection and analysis of data to predict future trends as a way to inform long-term decision-making.

Which workforce forecasting technique solicits the input of professionals who never actually meet? A. nominal group technique B. Delphi technique C. trend projection D. job bidding

The correct answer is B. The Delphi technique is a way to forecast future job needs by combining the input and expertise of many professionals who never meet, but who come to a consensus over time.

FASB was established in 1972 to do what? A. set guidelines for executive compensation B. set generally accepted accounting principles C. set ethical standards for corporate governance D. set standardized HR metrics

The correct answer is B. The Financial Accounting Standards Board (FASB) was established in 1972 by the American Institute of Certified Public Accountants, and charged with setting, maintaining, and updating the Generally Accepted Accounting Principles that are recognized by the federal government as the authority for accounting practices in the United States.

The ________ Act regulates the storage and retention of electronic communications like emails. A. Sarbanes Oxley B. Stored Communications C. Computer Fraud and Abuse D. Employee Privacy Policy

The correct answer is B. The Stored Communications Act, passed in 1986, protects the storage of electronic communications. In general, employers are not prohibited from accessing communications stored on their wire or electronic communications services (such as employer-provided email service). However, companies should announce this policy.

Nadia works in HR and wants to perform a compa-ratio on Robert's salary. If the midpoint of the company's salary range is $60,000 and Robert's pay is $78,000, what is his compa-ratio? A. 0.77 B. 1.3 C. 2.6 percent D. 1.77 percent

The correct answer is B. The compa-ratio allows for an employer to determine how close an employee's pay is to the middle of an organization's pay range. Since the midpoint is already given, divide Robert's salary by the midpoint to get 1.3; to express the compa-ratio as a percentage, multiply by 100 to get 130 percent.

In which of the situations would the correlation coefficient for compensation most likely be negative? A. a graphic designer is not paid a fixed salary B. an engineer must work longer hours to complete a project C. a sales representative receives commission for every sale she makes D. a manager is forced to take the morning shift, whereas before, she worked in the afternoon

The correct answer is B. The correlation coefficient is most likely negative for an employee who must give up some of their leisure time to increase their performance. Meeting a project deadline might warrant such a performance increase.

Which of the following is not one of the four key organizational characteristics of a business? A. structure B. personnel C. culture D. processes

The correct answer is B. The four key organizational characteristics of any business are structure, systems, culture, and processes. Personnel is an essential element of any business, but they are not an organizational characteristic.

Which of the following describes the attitudinal and organizational barriers that inhibit women from advancing in their careers? A. tolling B. glass ceiling C. four-fifths rule D. systemic discrimination

The correct answer is B. The glass ceiling refers to the attitudinal and organizational barriers that inhibit women from advancing in their careers.

Gloria, an Administrative Assistant at XYZ Corporation, overhears the head of the auditing department tell her assistant that the company will be reporting a record quarterly profit next week. She tells her brother Chip the news, and Chip buys 1,000 shares of XYZ stock, which soars in value after the record profit is announced; he gives Gloria 100 shares to thank her for giving him the information. Who in this scenario is not guilty of insider trading? A. Gloria, because she does not have a management role at XYZ Corporation B. the head of the auditing department, because she did not seek to profit from the information C. Chip, because he is not an employee of XYZ Corporation D. both Chip and Gloria, because they are by definition not "insiders"

The correct answer is B. The head of the auditing department did not try to profit from the inside information about XYZ record profit, though she should probably have been more discreet about discussing that information in a place where she could be overheard. Both Chip and Gloria are guilty of insider trading. It does not matter that Gloria is not in management or that Chip is not an employee; both profited in a stock transaction by acting on information that was not available to the general public.

Which managerial function involves providing a business with all the physical, financial, and human resources it will need to accomplish its organizational goals? A. Planning B. Organizing C. Directing D. Controlling

The correct answer is B. The organizing function involves providing an organization with needed resources, assigning duties, delegating authority, and coordinating authority and responsibility relationships.

What is the primary advantage of using global virtual teams? A. By using team members from lower-wage countries, global virtual teams can lower costs B. By drawing on a larger pool of workers, global virtual teams can leverage higher-level skills and competencies C. By drawing on a larger pool of workers, global virtual teams can increase diversity D. By operating in a virtual environment, global virtual teams can reduce overhead by eliminating the need for office facilities and business travel

The correct answer is B. The primary advantage of using a global virtual team is that it can draw upon a wide range of members with higher-level skills and competencies. Diversity, by itself, is not a central business goal. And any cost savings that might result from the use of global virtual teams is an ancillary benefit, not a primary advantage.

During which phase of project management is the timetable established? A. initiating B. planning C. executing D. closing

The correct answer is B. The timetable is established during the Planning phase.

According to Money Inc., Jay Leno owns 169 cars and 117 motorcycles. These data points are examples of ________. A. continuous data B. discrete data C. ordinal data D. ratio data

The correct answer is B. These measures of Leno's vehicular holdings are represented as whole values with clear boundaries - Leno does not own 168.7 cars or 117.3 motorcycles - and therefore are examples of discrete data.

The process of coordinating all the operating functions of a business including human resources, in support of the business's overall strategy is known as_______. A. Recruitment Strategy B. Strategic Alignment C. SWOT analysis D. PEST analysis

The correct answer is B. This describes strategic alignment, a process for ensuring that all operating functions of a business are aligned to support the business's overall strategy.

Richie is a captain on the Army Reserve; he takes a four-month leave of absence from his job to take mandatory training. Which statement describes his rights under USERRA after he returns from his leave of absence? A. Richie is entitled to back pay for the four months of work that he missed B. Richie can return to his old job, and he is given extra time to make up missed contributions to his 401k C. Richie cannot be fired without cause for one year D. Richie cannot be promoted for one year

The correct answer is B. Under USERRA, employees who must take a leave for military service are entitled to return to their old jobs. They do not receive back pay, but employees who leave for less than six months cannot be terminated without cause for six months, and their retirement benefits are protected.

When HR conducts orientation training for new employees, it is exercising what role? A. Advisory role B. Service role C. Control role D. Counseling role

The correct answer is B. When HR functions as a centralized staff to perform activities that serve multiple departments (such as recruiting, training, and record-keeping), it is exercising the service role.

Acme Corp. pays its computer programmers 12 percent less than programmers at other companies in the area. What sort of issues does this create? A. lack of internal equity B. lack of external equity C. inflated compa-ratio D. faulty job evaluation

The correct answer is B. When a company's pay levels are not comparable to those of nearby companies, there is a lack of external equity.

Which of the following is not an effective tactic for encouraging constructive debate within a team? A. having team members paraphrase each other's ideas B. downplaying differences among team members to encourage a sense of unity C. inviting constructive face-to-face feedback D. identifying differences within the team to achieve a common understanding of all perspectives embodied by the team

The correct answer is B. When working with a diverse team, it is important to recognize and acknowledge the differences in perspective that account for that diversity in order to best employ the insights each team member is able to provide thanks to their unique perspective.

Which of the following is not among the ethical responsibilities of an HR professional? A. advocating for the welfare of employees B. maximizing immediate profit for the organization C. protecting the long-term interests of the organization D. maintaining the organization's code of ethics

The correct answer is B. While increasing profit tends to be a part of advancing the long-term interests of the organization, it must be balanced as a goal with other ethical responsibilities; pursuing profit to the cost of all else is likely to lead to ethical violations that may, in the long term, harm the company far more than whatever benefit it derives from short-term financial gains.

Which of the following types of leadership theories is still in general use? A. Great Man B. situational C. trait-based D. behavior-based

The correct answer is B. While the Great Man theory of leadership has been largely discredited, and trait-based and behavior-based theories have given way to more advanced constructs, situational leadership theories remain the basis for much current thinking on the issue of leadership.

What is the greatest drawback involved in the use of virtual teams? A. faulty technology B. poor communication C. time differences D. higher cost

The correct answer is B. While the technology needed for remote work continues to advance, the need for clear, real-time communication is paramount. Time differences, while sometimes a concern, are not an insurmountable obstacle, and it is not clear that virtual teams necessarily cost more than co-located teams.

How should digital documents containing sensitive information be disposed of? A. by erasing them from the hard drive B. by overwriting their contents with random characters C. by performing a DoD 5220.22-M wipe D. by physically destroying the hard drive

The correct answer is C. A DoD 5220.22-M wipe is a highly effective method of data disposal that wipes a hard drive and overwrites all existing information with random junk characters. There are multiple free software programs available that will perform DoD 5220.22-M wipes.

What is a HiPO? A. an investor who could be approached for new financing B. an executive who could be awarded a significant raise C. an employee who could be groomed for a corporate leadership position

The correct answer is C. A HiPO, or high-potential employee, is someone who could be groomed for a corporate leadership position.

An ________ is software or an online system that is used for data entry, data tracking, and the data information requirements of an organization's HR management operation. A. ERP B. ICE C. HRIS D. EPP

The correct answer is C. A Human Resource Information System (HRIS) handles data entry, data tracking, and data information requirements for a company's HR operation. It is often a part of the larger, company-wide Enterprise Resource Planning (ERP) system.

A correlation coefficient measures the relationship that exists between two variables, but it does not demonstrate ________. A. duality B. induction C. causality D. deduction

The correct answer is C. A correlation coefficient provides a measure of how two variables are related, but it does not show whether either variable causes the other.

You have been tasked with conducting a culture audit of your organization, as the first step in a campaign to change the overall organizational culture. What is the best way to gather information for this project? A. employee survey B. confrence with top management C. in-person interviews with employees D. focus groups with customers

The correct answer is C. A culture audit focuses on the attitudes, beliefs, and customs of the organization's employees; as such, it does not take into account the views of customers. The audit should be extensive and organization-wide, not limited to the views of top management. Extensive in-person interviews, which allow for follow-up questions and detailed answers, will produce more and better information than a survey of employees.

The HR accounting measure that represents all costs associated with recruiting, selecting, training, and developing all the current employees of a firm is known as the ________. A. replacement cost B. separation cost C. outlay cost D. lost performance cost

The correct answer is C. A firm's outlay cost is the total cost the firm incurred in recruiting, selecting, training, and developing all the current employees of a firm. Replacement cost is the estimated cost to replace all existing employees at current expense rates, plus the opportunity costs of lost income while the replacement employees are developing the same level of proficiency as the employees they replaced.

Which of the following is a major barrier for change initiatives? A. technology issues B. employees' fear of automation C. employees' resistance to change D. employers' lack of enthusiasm

The correct answer is C. A major barrier to change initiatives is employees' reluctance to adopt new ways and methods.

David McClelland is most closely identified with which of the following theories? A. Expectancy Theory B. Two-Factor Theory C. Theory of Acquired Needs D. Hierarchy of Needs Theory

The correct answer is C. David McClelland developed the Theory of Acquired Needs, also known as the Three Needs Theory, in the 1960s.

A good or service developed for the client during a project is known as a _____. A. scope B. executable C. deliverable D. closing

The correct answer is C. Deliverables are the goods or services developed during a project, which are to be delivered to the client.

Which of the following is an example of a lagging indicator? A. time to fill key staff vacancies B. turnover rate C. gross margin D. average cost per hire

The correct answer is C. Gross margin is a lagging indicator—that is, an output-oriented metric that measures the results of a process or change. In this case, gross margin provides one measure of the company's profitability.

Which of the following is not an example of a vision statement? A. "Massachusetts Eye and Ear will be the preeminent world-wide source of advances and leaders in preserving and restoring vision, hearing, balance, and voice, as well as in curing disorders of the head and neck." B. "Be recognized by the people and organizations we serve, as well as others in our field, as the provider of choice for blood, plasma, and tissue services. This will be accomplished by commitment to quality, safety, and use of the best medical, scientific, manufacturing, and business practices." C. "The American Red Cross, a humanitarian organization led by volunteers, guided by its Congressional Charter and the Fundamental Principles of the International Red Cross Movement, will provide relief to victims of disasters and help people prevent, prepare for, and respond to emergencies." D. "Students whose lives are impacted by FFA and agricultural education will achieve academic and personal growth, strengthen American agriculture and provide leadership to build healthy local communities, a strong nation, and a sustainable world."

The correct answer is C. A vision statement describes what an organization seeks to accomplish in the years ahead; it lays out the organization's aspirational goals for the future. A mission statement, by contrast, spells out what the organization does in the day-to-day present. Choices A, B, and D all speak of what their respective organizations will be in the future; choice C spells out the mission of the American Red Cross, which is to provide emergency relief to victims of disasters.

Brenda is asked to prepare an analysis of personnel needs for the next year, based on an expected 7 percent increase in output. Brenda determines the type and amount of work that will be needed to meet that goal, then breaks that work down into man-hours. Finally, she translates total additional man-hours into the number of new workers the company will need to hire. This is an example of ________. A. unit demand B. trend projection C. workload analysis D. ratio analysis

The correct answer is C. A workload analysis is a short-term needs analysis that begins with the company's goals for the immediate future; it then works backward to calculate the man-hours needed to achieve that goal, and then translates man-hours into personnel needs.

Which of the following would not be an effective strategy for encouraging healthy debate among team members? A. asking them to paraphrase each other's ideas B. inviting feedback focused on the facts supporting each argument C. encouraging team members to openly share any personal conflicts that might inhibit productive discussion D. dividing speaking time equally among all team members

The correct answer is C. Allowing a debate to stray into matters of personal conflict, rather than staying focused on the issues at hand, will usually impede constructive debate.

Which of the following is a major goal of conducting an HR audit? A. to assess the organizational culture B. to assess the HR department's budget C. to assess compliance risk D. to assess the effectiveness of HR policies

The correct answer is C. An HR audit is designed to see which HR policies are functioning well, which are not functioning well, and which HR policies may be out of compliance with legal requirements. An HR audit is a critical element in enterprise risk management (ERM).

Which of the following is typically not included in an RFP? A. a description of the project B. a description of the organization that is sponsoring the RFP C. a description of the organization's products and services D. a description of the process for submitting a proposal

The correct answer is C. An RFP should include a description of the project, a description of the proposal process, and a brief description of the organization that is sponsoring the RFP, to help potential vendors in preparing an accurate bid. It should also include a description of the products and/or services the vendor is being asked to provide, but it would not normally include a description of the products and services that the sponsoring organization sells; that information is not relevant to the RFP process.

Both sides to a dispute agree to accept the decision of a neutral third party. A. mediation B. conciliation C. arbitration D. constructive confrontation

The correct answer is C. Arbitration is a dispute-resolution procedure under which both sides agree to accept the decision of a neutral third party.

In ________, a neutral third party is invited into deadlocked negotiations to make decisions which both negotiating parties agree to abide by. A. mediation B. moderation C. arbitration D. constructive confrontation

The correct answer is C. Arbitration is the use of a neutral third party, an arbitrator, invited into negotiations by both contending parties to resolve an impasse.

Five months after an intensive training program for its IT staff, a company wants to know if the employees are actually using the knowledge and skills they received. Which level of evaluation should the company utilize? A. reaction B. learning C. behavior D. results

The correct answer is C. Behavior measurements assess job performance after training takes place.

In some foreign countries, bribery and kickbacks are widely viewed as a culturally accepted way of facilitating business agreements. Which of the following should control a U.S. company's decision about whether to engage in bribery as a foreign business practice? A. the laws of the foreign country involved B. the United Nations Charter C. U.S. law D. all of the above

The correct answer is C. Bribery of foreign nationals is explicitly illegal under the U.S. Foreign Corrupt Practices Act of 1977. Bribery most likely also violates the laws of the country involved, but this would not be a controlling factor in a country where bribery is an accepted business practice. And, though the General Assembly adopted the United Nations Convention against Corruption in 2003, the UN Charter itself makes no mention of bribery, kickbacks, or corruption.

Which of the following statements is true? A. Organizational climate is enduring, while organizational culture is changeable. B. Organizational climate is unrelated to organizational culture. C. Organizational culture is enduring, while organizational climate is changeable. D. Organizational culture is the product of organizational climate.

The correct answer is C. Climate and culture are closely related: culture is the enduring set of beliefs that guide the behavior of an organization and its members, while climate is the more short-term set of attitudes that members hold about the organization at any point in time.

Computer hacking is a direct threat to which of the following: A. human assets B. financial assets C. information assets D. communication assets

The correct answer is C. Computer hacking is a prominent threat to information assets

Which of the following situations best describes a manager whose leadership style is Democratic? A. Satoe offers her employees bonuses when they meet objectives. B. Satoe assigns a project to her team and waits for them to finish it. C. Satoe allows her team to make some decisions that will affect the outcome of a project D. Satoe keeps her desk organized and encourages team members to collaborate on projects.

The correct answer is C. Democratic approaches to leadership are most appropriate when employees are highly skilled and can take an active role in decision-making processes. If Satoe is completely hands-off, as in the second option, she most likely utilizes a Laissez-Faire style of leadership. A Democratic approach lets employees in on the big picture and allows them to be active in the decision-making process.

Which of the following is most clearly identified with the concept of the quality circle? A. Joseph Moses Duran B. William Edwards Deming C. Kaoru Ishikawa D. Philip B. Crosby

The correct answer is C. Dr. Kaoru Ishikawa helped introduce the concept of the quality circle in the early 1960s.

Damian is going to conduct interviews with two job candidates, Rita and Kimberly. Since both have experience in the same field, Damian decides he will ask them about their respective job histories and how they handled difficult situations in the past. Which type of interview will Damian most likely conduct? A. directive interview B. patterned interview C. behavioral interview D. non-directive interview

The correct answer is C. During behavioral interviews, candidates are asked to explain how they responded to problems or events in the past. The assumption is that these past behaviors will predict how employees would respond to similar situations in the future. This can be a good option when the candidates' resumes are similar.

Which of the following best describes Ning, a Senior Systems Analyst who is in the disengagement stage of the career lifecycle? A. Ning is still figuring out if he wants to go to law school or not. B. Ning's productivity is on the rise after a recent promotion to senior systems analyst. C. Ning recently got a job offer with another company that comes with more vacation time. D. Ning is taking a new course to specialize his skills through the company's development program.

The correct answer is C. During the disengagement stage, individuals start to pull away from their work for a variety of reasons, one of which can be receiving a better job offer. Ning may find the extra vacation time more appealing and begin to lose interest in his current work.

Gabriele is the COO at Gwynplaine Media, a video production company. She got into video production 25 years ago because she loves telling stories on the silver screen. She quickly rose through the ranks at Gwynplaine and has been there for 15 years. Lately, however, she's been disinterested in her work. The CEO suggests that she mentor Adam, a competent Development Executive who is ready for more responsibility. What stage in the career lifecycle is Gabriele most likely at? A. exploration B. establishment C. maintenance D. disengagement

The correct answer is C. During the maintenance stage, an individual's performance may stagnate because they have reached a plateau in their career. One way to get the individual to become reinvested in their career is to serve as a mentor.

An employee's talents are typically optimized at what stage of the career life cycle? A. exploration B. establishment C. maintenance D. disengagement

The correct answer is C. During the maintenance stage, employees reach a level at their organizations where their talents are optimized, which can mark a plateau.

Which is one major purpose of employee self-service (ESS) technology? A. It allows managers easier access to performance management records B. It allows job candidates to input their applications and resumes C. It allows employees to update their personal information D. It allows senior management to track HR performance

The correct answer is C. ESS technology, which is a basic functionality of most large HCM systems, allows employees to update their personal information, apply for reimbursement, and access information about their company benefits.

Which of the following is the ability to identify and control one's emotions to achieve positive outcomes in their relationships? A. social quotient B. social awareness C. emotional intelligence D. performance appraisal

The correct answer is C. Emotional intelligence is one's ability to identify and control their emotions to achieve positive outcomes in their relationships.

Managers focused on providing a healthy work-life balance to their employees would most likely do which of the following? A. listen to employees' feedback B. offer professional development training C. encourage employees to take vacation time D. establish a formal system for awarding bonuses

The correct answer is C. Encouraging employees to take their vacation time and not be available 24/7 signals to workers that the organization understands the need for a healthy work-life balance.

The stage of a SWOT analysis in which the company assesses its business environment to identify opportunities and threats is known as ________. A. environmental analysis B. environmental testing C. environmental scanning D. environmental assessment

The correct answer is C. Environmental scanning is the second step of a SWOT analysis.

Which of the following refers to the probability that effort will lead to performance? A. equity B. valence C. expectancy D. instrumentality

The correct answer is C. Expectancy refers to the probability that effort will lead to performance.

Which of the following is an example of an extrinsic reward? A. taking pride in a job well done B. the feeling that you're getting better at your job C. being chosen "employee of the month" D. the sense that your job fulfills a meaningful purpose

The correct answer is C. Extrinsic rewards are those bestowed by others, such as being chosen as "Employee of the Month."

What type of organizational structure is best suited for chaotic environments filled with complex relationships and rapid change? A. institutional organization B. entrepreneurial organization C. flexible organization D. proprietorship organization

The correct answer is C. Flexible organizations, which feature decentralized decision-making, are best suited for dealing with complex environments and rapid change. Proprietorship organizations are the opposite; they operate best in a simple environment marked by relative stability. Entrepreneurial organizations also function best in a simple environment but are able to deal with a more rapid rate of change. Institutional organizations can deal with complex environments, but their bureaucratic structure means they have a harder time dealing with rapidly changing circumstances.

What type of Congressional committee is used to reconcile the differences when the House and Senate each pass different versions of a bill? A. standing committee B. select committee C. conference committee D. steering committee

The correct answer is C. If the House and Senate pass different versions of the same bill, and neither house will agree to accept the other's version, a conference committee is used to resolve the differences. The leaders of each house appoint conferees (usually senior members of the legislative committees that originally handled the bill in each chamber), who then meet in committee to negotiate a compromise bill that each house can accept.

Randall, an HR Manager at Johnson Chemicals, is considering options to increase the diversity of the company's middle managers. Which of the following initiatives would be least effective in pursuing that goal? A. target recruiting efforts at underrepresented communities B. focus less on candidates' academic credentials, in favor of specific skills C. intensify the company's promote-from-within policy D. partner with professional groups that represent diverse communities

The correct answer is C. If the company already has a problem with diversity in its middle-management ranks, doubling down on internal promotions will make the problem worse, not better.

Within the SMART model, the letters stand for Specific. ________, Action-Oriented, Relevant, and Time-bound. What does the "M" stand for? A. mature B. metric C. measurable D. metadata

The correct answer is C. In the SMART model, the "M" stands for Measurable. The other options (Mature, Metric, and Metadata) do not fit into the SMART model, which is used to help a company define its long-term strategic goals.

Which of the following is NOT one of Michael Porter's five forces that influence the success of a business? A. New Competitors B. Suppliers C. Independent Contractors D. Competition in the industry

The correct answer is C. Independent contractors are not one of the five forces identified by Porter.

Which of the following best describes Insight, one of the Six I's of Innovation? A. Take the necessary steps to alter or sustain current actions and behaviors. B. Articulate the specific actions and behaviors that are necessary for achievement of its goals. C. Develop multiple ways of problem-solving for future issues, such as brainstorming and reverse engineering. D. Periodically perform a structured assessment of whether the people responsible for those actions and behaviors are functioning as desired.

The correct answer is C. Insight, one of the I's of Innovation, can be encouraged by developing multiple ways to problem-solve. The other three options describe a performance management system.

Jaime Guzman, director of HR for a regional supermarket chain, is tasked with determining whether labor costs in the southern part of the region are out of line with the regional average. Toward that end, he tracks total compensation as a percentage of total sales for each outlet in the chain, and then compares the average data for the 12 stores in the southern region against data for all 37 stores in the chain. This is an example of: A. needs assessment B. tactical analytics C. internal benchmarking D. external benchmarking

The correct answer is C. Internal benchmarking involves making comparisons to other periods in a company's history, or between outlets or divisions within the same company. External benchmarking involves making comparisons between an organization and similar organizations. Needs assessment is an effort to make predictions about an organization's future needs and productivity, and tactical analytics involves the use of data to address real-time HR issues.

The CEO of Hurlatron Plastics is concerned that employee salaries are outstripping employee productivity. Adele Metzler, director of HR at Hurlatron, assembles data that show total compensation as a percentage of net income before taxes over the last five years, and finds that this figure has not increased over time. This is an example of: A. needs assessment B. tactical analytics C. internal benchmarking D. external benchmarking

The correct answer is C. Internal benchmarking involves making comparisons to other periods in a company's history, or between outlets or divisions within the same company. External benchmarking involves making comparisons between an organization and similar organizations. Needs assessment is an effort to make predictions about an organization's future needs and productivity, and tactical analytics involves the use of data to address real-time HR issues.

Stacey is an entry-level employee who displays strong communication and social skills in her interactions with coworkers. While Amelia, Stacey's manager, prepares Stacey's first performance review, she determines that Stacey has struggled twice to meet minor deadlines and has not mentioned what her long-term goals are. During the review, Amelia asks Stacey if she has a five-year plan, to which Stacey answers no. Which of the following should Amelia encourage Stacey to work on? A. social intelligence B. emotional intelligence C. intrapersonal intelligence D. interpersonal intelligence

The correct answer is C. Intrapersonal intelligence refers to the ability to self-reflect and to manage and plan one's life. Stacey would most likely benefit from developing it in herself.

Which of the following best describes job bidding? A. a manager submits a request to add a new position B. an employee submits a letter of resignation C. an employee expresses interest in a position before the position is available D. an employee receives training for a future position

The correct answer is C. Job bidding occurs when a current employee expresses interest in a position before it is available to be filled. This can be an effective way for an organization to fill vacancies with internal candidates.

Which of the following is an argument in favor of affirmative action? A. Today's workers should not have to pay to right the wrongs committed many years ago B. Many racial minorities and women resent being advanced in life due to their race or sex, rather than their merit C. The interests of white men need to be balanced against society's interest in promoting affirmative action programs D. Affirmative action is itself a form of discrimination--disparate treatment of white men

The correct answer is C. Many advocates of affirmative action cite the need to balance the interests of white men against the interests of society as a whole.

Which of the following is most likely the result of intentional turnover? A. A COO in her late 30s leaves after 10 years with the company. B. A manager in her late 50s leaves after six years with the company. C. A coordinator in his early 30s leaves after two years with the company. D. A sales representative in his early 40s leaves after five years with the company.

The correct answer is C. Many experts believe that if an employee leaves the company within one to three years, then most likely the company is going through intentional turnover or has onboarding issues.

An organization decides to implement a change due to internal needs. The leadership convinces employees that the change is necessary and beneficial. The employees assure the leadership that they are making an earnest effort to change, but after two attempts at implementation, it does not produce the desired results. In which stage of Lewin's theory is the organization stuck? A. unfreezing B. training C. moving D. refreezing

The correct answer is C. Moving includes implementing the change, making needed revisions, and reinforcing the change. If the employees are making an effort to change and the change is not producing the desired results, then revisions are needed.

Which federal agency is responsible for enforcing the whistle-blower protections of the Sarbanes Oxley Act? A. Department of Justice B. SEC C. OSHA D. IRS

The correct answer is C. OSHA, a division of the Department of Labor, is charged with enforcing the whistle-blower protections of the Sarbanes Oxley Act.

Under which of the following conditions can picketing be ruled illegal? A. if it produces a threatening environment B. if it disrupts traffic C. both A and B D. neither A nor B

The correct answer is C. Picketing in large numbers, known as mass picketing, can become illegal if it produces a threatening environment or disrupts traffic. It can also become illegal if it physically impedes people from entering or exiting the picketed location.

Jenna is a Sales Representative and receives her healthcare through her employer, which has a contract with JRB, a health insurance company. It is a group health plan that allows Jenna to go to any physician in her network without a referral, but if she goes outside her network, she must pay a higher rate. Which of the following types of healthcare plan does Jenna most likely have? A. fee for service B. point of service C. preferred provider organization D. health maintenance organization

The correct answer is C. Preferred provider organizations are contractual arrangements between healthcare providers and an employer or health insurance company, which allow subscribed members wide latitude to visit any physicians within the organization's network of providers. Out-of-network care is available but at a higher price.

What is the major advantage of product departmentalization? A. It is the most efficient way for small companies to organize themselves. B. It promotes greater skill specialization. C. It promotes greater cooperation between parts of the company. D. It eliminates the duplication of equipment and effort.

The correct answer is C. Product departmentalization promotes greater cooperation between different parts of the company as they focus on the production of a superior product, resulting in a company that is more customer-focused.

Which of the following is not an appropriate way for HR to support corporate governance? A. monitor compliance with all applicable legislation and regulations B. align HR procedures with corporate governance policies C. revise the organization's mission and vision statements as needed D. advise the board on making sure that governance policies include important HR issues

The correct answer is C. Revising the organization's mission and vision statements is in the purview of the board of directors and senior management, not HR.

Eve is involved with short-range planning. Which of the following do her decisions most likely affect? A. forecasting B. attrition rate C. day-to-day activities D. meeting organizational objectives

The correct answer is C. Short-range plans will typically have a lot of influence on day-to-day activities.

Two ways to increase employee involvement. A. town-hall meetings and task forces B. unions and committees C. suggestion boxes and delegation of authority D. newsletters and surveys

The correct answer is C. Suggestion boxes and delegation of authority are both ways to increase employee involvement.

Which of the following statements about employee feedback surveys is false? A. Surveys tend to be low- or no-cost. B. Surveys are easy to design using free or inexpensive platforms. C. Surveys typically provide qualitative data. D. Surveys can be repeated easily.

The correct answer is C. Surveys typically provide quantitative data.

Deena is an HR Manager examining two potential candidates. To perform a standard background check, Deena will most likely view all of the following documents EXCEPT _________. A. credit history B. motor vehicle records C. tax returns D. high school or college transcripts

The correct answer is C. Tax returns are not part of a standard background check.

Yesterday's high temperature was 64.5 degrees Fahrenheit. This measure is an example of ________. A. nominal data B. ordinal data C. interval data D. ratio data

The correct answer is C. Temperature is an example of interval data: the difference between each degree on the Fahrenheit scale is equal and meaningful, zero degrees Fahrenheit is an arbitrary rather than natural value, and it does not signify the absence of heat.

Which of the following is NOT a provision of the Fair Labor Standards Act? A. nonexempt employees must be paid overtime B. minors under the age of 18 are not allowed to work in hazardous jobs C. nonexempt employees must be paid double-time for working on Sundays D. employers must keep records of wages paid, hours worked, and other relevant data for each nonexempt employee

The correct answer is C. The Fair Labor Standards Act does not mandate extra pay for work on Sundays or holidays.

Which of the following HR metrics represents the average cost of providing HR services to the employees in a company? A. HR-to-Employee ratio B. cost per hire C. HR Expense Factor D. Human Capital ROI

The correct answer is C. The HR Expense Factor, calculated as HR Expenses / Total Employees, represents the average cost of providing human resource services to the employees in a company.

Terrence, an HR manager, was just told that the company is hiring Allen, a new Sales Representative who has HIV-AIDS. Allen is the first known employee to have HIV-AIDS, and knowledge of his condition will be made aware to all employees with Allen's consent so that he may receive reasonable accommodations. What should Terrence do in the near future? A. set up an employee assistance program B. ask Allen to take regularly scheduled medical exams C. educate all employees how AIDS is transmitted D. train the Sales manager to evaluate whether Allen can perform essential job junctions

The correct answer is C. The company must educate all employees on how AIDS is and is not transmitted.

Which of the following should be the first step in investigating alleged employee misconduct? A. interview the person who raised the allegation B. interview the person being accused of misconduct C. determine exactly what behavior is being investigated and what evidence is needed D. specify individuals who need to be interviewed

The correct answer is C. The first step in investigating any allegation of misconduct should be to clearly specify what behavior or conduct is being investigated and what evidence is needed to prove guilt.

Which of the following is not one of the functions of management? A. Planning B. Directing C. Funding D. Organizing

The correct answer is C. The five functions of management are Planning, Organizing, Staffing, Directing, and Controlling.

The ____ measures the gross (earned) income form a company's new products or services, compared to total gross (earned) income. A. Opportunity conversion rate B. Turnover rate C. Innovation rate D. Customer acquisition cost

The correct answer is C. The innovation rate measures the gross (earned) income from a company's new products or services, compared to total gross (earned) income. This metric shows the profitability of innovation from within an organization, and it is often compared to the innovation rates at other competing organizations.

Marcus, an IT employee who has worked at the same firm for two years, is unfamiliar with a new program that the company is adopting. Which of the following will highlight the gaps in Marcus's knowledge base? A. development B. implementation C. needs assessment D. change management

The correct answer is C. The needs assessment should surface whatever gaps exist between Marcus's knowledge base and what is required for the new program.

The practice of establishing a small number of broadly defined pay levels is known as____. A. broadsiding B. broadsheeting C. broadbanding D. compa-ratio

The correct answer is C. The practice of establishing a small number of broadly defined pay levels is known as broadbanding.

The federal agency that oversees union elections. A. NLCS B. NTSB C. NLRB D. NFLPA

The correct answer is C. The task of overseeing union elections falls to the National Labor Relations Board (NLRB).

What is a "matrix tax?" A. the increased cost for management in maintaining two different lines of authority in a matrix structure B. the increased cost to consumers for products manufactured by a matrix organization, which is less efficient than traditional structures C. the increased stress and anxiety for workers who are assigned to more than one department in a matrix structure D. the increased cost of salaries in a matrix structure, because of the duplication of management positions

The correct answer is C. The term "matrix tax" refers to the increased demands placed on workers (in terms of ambiguity, stress, and anxiety) who are assigned to more than one department in a matrix organization.

Moriarty International recently outsourced the administration of its employee benefit plans. As Moriarty's HR Director, how would you measure the business impact of the outsourcing? A. calculate the total cost savings achieved by the outsourcing B. calculate benefit expenses as a percent of total revenue C. compare employee satisfaction with benefits against the total cost savings achieved by the outsourcing D. compare employee satisfaction with benefits against productivity per employee

The correct answer is C. The total business impact of the outsourcing is not simply the cost savings achieved; you must apply a cost-benefit analysis that weighs the cost savings against the impact of this move on employee satisfaction, which in turn can affect turnover, productivity, and related factors.

Which of the following is NOT usually part of a company's affirmative-action program? A. utilization analysis B. removing inappropriate barriers to employment C. hiring quotas D. preferential hiring

The correct answer is C. The use of specific and inflexible hiring quotas is a violation of federal law.

At Oswald's Umbrellas, employees are compensated based on the company's overall performance. How much an employee is paid depends on their level of responsibility, seniority, and what their base pay is. Which type of payment system does Oswald's Umbrellas utilize? A. merit pay B. commission C. profit sharing D. group incentive pay

The correct answer is C. This company utilizes profit sharing. How the profits are divided is determined by an allocation formula, which usually takes into consideration how much responsibility an employee has, their seniority, and base pay.

The midpoint for a particular pay grade at Walsh Technologies is $60,000. What would you expect to be the upper and lower limits for this pay grade? A. upper limit of $80,000 and lower limit of $40,000 B. upper limit of $63,000 and lower limit of $57,000 C. upper limit of $69,000 and lower limit of $51,000 D. upper limit of $90,000 and lower limit of $30,000

The correct answer is C. Typically, pay grades have upper and lower limits that are 10 to 20 percent above and below the midpoint. Choice C falls into that range, with upper and lower limits that are 15 percent above and below the midpoint. All of the other choices are either too narrow or too broad.

At the request of the board of directors, the HR department of Megacorp has set goals and timetables that the company's manufacturing division will need to meet in order to achieve its affirmative action goals. What role is the HR department exercising? A. advisory role B. service role C. control role D. counseling role

The correct answer is C. When HR is tasked to implement policies and procedures and monitor compliance with them, it is exercising the control role.

Which of the following may cause employees to resist change? A. Cooperation B. Coordination C. Compulsion D. Contrition

The correct answer is C. When managers try to compel employees to accept change, employees often resist out of a sense of resentment.

Which of the following legal principles is a major factor in determining the validity of workplace searches? A. Fruit of the poisoned tree B. Unreasonable search and seizure C. Reasonable expectation of privacy D. Exclusionary rule

The correct answer is C. Whether a workplace search is found to be lawful depends in large part on whether the court finds that the worker did or did not have a reasonable expectation of privacy on the job. Businesses that have a clearly stated and widely circulated privacy policy can effectively determine what the worker may reasonably expect, in terms of his right to privacy in the workplace. Choice B, unreasonable search and seizure, refers only to searches or seizures conducted by the government, which are prohibited by the Fourth Amendment. Choices A and D (fruit of the poisoned tree and exclusionary) are legal concepts related to the admissibility of evidence obtained in violation of the Fourth Amendment.

Which of the following statements best characterizes the security of buildings located near natural features such as woods, rivers, lakes, etc.? A. They are usually less secure. B. They are usually more secure. C. They may be either more or less secure than buildings in other places. D. The presence of such natural features makes no difference to building security.

The correct answer is C. While buildings adjacent to rivers, lakes, and other obstructive features may benefit from extra "moat" security, other features, such as woods, may provide cover to would-be thieves approaching the building.

Bert is compiling a listing of all the KSAs and other attributes that are needed to perform the job of production foreman at Swanson Industries. When he is finished, Bert will have completed a ________. A. job description B. job analysis C. job evaluation D. job specification

The correct answer is D. A job specification is a listing of all the knowledge, skills, abilities, and other attributes needed to perform a specific job properly.

Which of the following is an example of laissez-faire leadership? A. Randy sets specific goals for members of his team and offers rewards to all who achieve those goals. B. Sarah serves as a role model for her department, encouraging collaboration and helping team members perform at a higher-than-expected level. C. Francisco places a premium on productivity, assigning each of his team members a specific task that needs to be completed, based on their particular skills. D. Jessica lets each of her reports decide where they can do best for the department, and she tells them to work at their own pace.

The correct answer is D. A laissez-faire leader like Jessica is one who exercises little control over her reports and allows them a great deal of freedom to operate. Sarah, by contrast, is a transformational leader - one who leads by inspiration, serving as a role model and transforming her workers by getting them to outperform expectations. Randy, who offers rewards in exchange for achievement, is a transactional leader. And Jesus, with his focus on productivity and his insistence on controlling work assignments, is a classic authoritarian or directive leader.

A tactic not employed by labor unions. A. picket line B. boycott C. slowdown D. lockout

The correct answer is D. A lockout, by definition, is a tactic employed by management, not labor.

Which of the following statements regarding employee performance rating scales is true? A. All organizations use the same standardized rating scales for employee performance. B. Rating scales are always quantitative. C. Rating scales directly compare one employee's performance to another's. D. Rating scales evaluate performance for each employee in multiple categories related to their work.

The correct answer is D. A rating scale defines categories of performance related to an employee's responsibilities and assigns that employee a rating for each category, which may be quantitative or qualitative.

Which contractor is exempt from an affirmative action program (during non-emergency times)? A. contractor with 55 employees who receives a federal contract for $55,000 B. contractor with 50 employees who receives a federal contract for $50,000 C. contractor with 75 employees who receives a federal contract of $60,000 D. contractor with 45 employees who receives a federal contract of $100,000

The correct answer is D. According to Executive Order 11246 and Section 503 of the Rehabilitation Act of 1973, federal contractors, with a minimum of 50 employees and at least one contract of $50,000, must have an affirmative action program. A contractor with only 45 employees who receives a federal contract does not meet these qualifications.

Which of the following terms describes an employee's annual bonus? A. family-oriented benefit B. intrinsic reward C. base rate D. variable compensation

The correct answer is D. An annual bonus is a form of variable compensation.

The best ethics codes are ________. A. as comprehensive and specific as possible B. consistent and unchanging over time C. written in the same formal style as typical business communications D. as simple and concise as possible

The correct answer is D. An ethics code is less useful if it is confusing, overly complex, or hard to remember fully. Codes written in complex detail can also be open to misinterpretation. The most effective ethics codes keep their language simple, concise, and clear.

The sense of satisfaction that comes from doing your job well done is an example of what? A. creative benefit B. psychic income C. extrinsic reward D. intrinsic reward

The correct answer is D. An intrinsic reward is one in which a person's esteem is generated within herself; satisfaction in a job well done is a classic example of an intrinsic reward. Extrinsic rewards come from others (e.g., praise from a supervisor or a raise in salary). Psychic income is a non-financial benefit, but it can be either intrinsic or extrinsic; similarly, a creative benefit is a non-traditional benefit or perk, which can be either intrinsic or extrinsic.

Astronauts rehearse their spaceflights extensively in simulators and centrifuges, and on parabolic flights that mimic weightlessness. This is an example of___. A. on-the-job training B. apprenticeship C. mentoring D. vestibule training

The correct answer is D. Astronaut training is a highly advanced (and extremely expensive) form of vestibule training.

Which of the following metrics can be used to help assess the efficiency of your company's marketing department? A. revenue per employee B. gross margin C. churn rate D. customer acquisition cost

The correct answer is D. Customer acquisition cost (CAC) is a measure of the total amount the company spends on its marketing efforts over a set period of time, divided by the number of customers it acquired over that same period of time. The CAC is then compared to the lifetime value of a customer to gauge whether the marketing department is spending too much or too little to acquire new customers.

Which of the following is an example of public data? A. biographical data B. a manager's qualifications C. the university from which an employee graduated D. the top executives of a competitor

The correct answer is D. Easily accessible information about a competitor is a type of public data.

How are exit interviews and post-exit surveys used in workforce forecasting? A. they are used as the basis of replacement charting B. they are used as the basis of a job bidding process C. they are used as the basis of workload analysis D. they are used as the basis of turnover forecasting

The correct answer is D. Exit interviews and post-exit surveys help HR professionals learn why employees are leaving the company, and this is a key factor in turnover forecasting.

Which of the following is not a tool used in turnover forecasting? A. focus group B. exit interview C. post-exit survey D. organizational audit

The correct answer is D. Focus groups, exit interviews, and post-exit surveys are all frequently used in turnover forecasting, but an organization audit is a tool used in strategic planning.

Which of the following is NOT an HR metric? A. Cost Per Hire B. Turnover Rate C. Training Per Employee D. Gross-Profit Margin

The correct answer is D. Gross-profit margin is a sales metric, not an HR metric.

Which of the following conditions is not necessary for an employee to claim age discrimination? A. The employee was fired or demoted. B. The employee is at least 40 years old. C. The employee has been performing the job well. D. The employee has been the victim of age harassment.

The correct answer is D. Having experienced age harassment is not a necessary condition to claim age discrimination.

What is the difference between inentives and benefits? A. incentives are always monetary; benefits are always non-monetary B. incentives are a form of compensation; benefits are not C. incentives are legally required; benefits are not D. incentives are meant to encourage performance; benefits are meant to enhance employee participation in the workplace

The correct answer is D. Incentives are designed to motivate employees to better performance, while benefits are designed to enhance their participation in the workplace by improving their economic security and work-life balance.

A present obligation of the organization, arising from past events, is known as ________. A. equity B. an asset C. income D. a liability

The correct answer is D. Liabilities are financial obligations that the organization has already incurred.

Why is it important for HR to enforce the company's compliance with applicable laws and regulations? A. Because failure to comply with laws and regulations exposes the company tp potentially costly legal action B. Because failure to comply with laws and regulations is morally and ethically wrong C. Because failure to comply with laws and regulations gives the company an edge in recruiting new employees D. A and B, but not C

The correct answer is D. Non-compliance creates legal liabilities and is morally and ethically wrong, but it does not usually give the company a competitive advantage in hiring.

Joan is interviewing candidates for an open position in the Accounting department. She is impressed by several of the candidates, but she has a strong negative reaction to one of them. This candidate has an impressive resume and solid recommendations, but Joan is put off by his habit of frequently cracking his knuckles. Joan finds this behavior annoying and eliminates the candidate from consideration. Which of the following interview biases is Joan demonstrating? A. first impression B. cultural moise C. gut feeling D. nonverbal bias

The correct answer is D. Nonverbal bias takes place when an interviewer is influenced by body language, behavior, or appearance.

During which phase of project management are the outcomes and processes evaluated? A. initiating B. planning C. executing D. closing

The correct answer is D. Outcomes and processes are evaluated during the Closing phase.

Which of the following is NOT examined during an organizational audit? A. Competence B. Governance C. Leadership D. Payroll

The correct answer is D. Payroll is not an organizational characteristic, and it is not examined during an organizational audit

In the middle of the day, a relatively minor earthquake with a magnitude of 3.0 occurs. Which of the following asset types is most likely to be harmed by the earthquake? a. financial assets B. information assets C. human assets D. physical assets

The correct answer is D. Physical assets include any equipment or property owned by an organization; such assets are the most likely to be harmed by an earthquake.

Which of the following is NOT part of a job description? A. title B. terms of employment C. reporting relationship D. required experience

The correct answer is D. Required experience is part of a job specification, not a job description.

Why is it important for companies to follow written procedures for progressive discipline? A. failure to follow written procedure for progressive discipline is a violation of the Fair Labor Standards Act B. failure to follow written procedures for progressive discipline creates confusion and the potential for bureaucratic error C. failure to follow written procedures for progressive discipline is a violation of the Wagner Act D. failure to follow written policies for progressive discipline may render an employee's dismissal invalid in court

The correct answer is D. Several court decisions have ruled that failure to follow written policies for progressive discipline may render an employee's dismissal invalid.

Which of the following events is most likely to lead to major changes in an organization's culture? A. culture audit B. Internal public offering C. recommendation of the board of directors D. death of the founder

The correct answer is D. Significant cultural changes are not usually planned or dictated by management consultants or the board of directors; they most often follow sudden or cataclysmic events that "unfreeze" the culture, such as the death or retirement of the founder, or a decision to merge or sell the company.

The process of coordinating all the operating functions of a business in support of the business's overall strategy is called ________. A. strategic planning B. strategic development C. strategic deployment D. strategic alignment

The correct answer is D. Strategic alignment is the process of coordinating all the operating functions of a business, including human resources, in support of the business's overall strategy. Effective alignment needs to take place at all levels of the organization; the strategies of individual business units need to align with the overall corporate strategy, but also with the strategies and tactics of smaller functional sub-units.

Supervision, motivation, leadership, and communications are elements of which management function? A. Planning B. Organizing C. Controlling D. Directing

The correct answer is D. Supervision, motivation, leadership, and communications are all elements of the Directing function.

Brenda has an hour-and-a-half commute back and forth to work each day. She asks her boss why she isn't compensated for this work-related time. Which federal statue does her boss cite? A. Davis Beacon Act B. Fair Labor Standards Act C. Walsh Healey Act D. Portal-to-Portal Act

The correct answer is D. The Portal-to-Portal Act, which specifies the types of compensable time for which employers are accountable, states that commute time is not compensable.

Due to an economic downturn, leaders of an organization have decided to layoff 200 of their 500 full-time employees. Which of the following laws requires that the employer provide at least 60 days' notice? A. National Labor Relations Act B. Labor-Management Relations Act C. Consolidated Omnibus Reconciliation Act D. Worker Adjustment and Retraining Act

The correct answer is D. The Worker Adjustment and Retraining Notification (WARN) Act requires employers with 100 or more full-time employees to provide 60 days' notice to employees or their union before going through with a mass layoff or plant closing. The WARN Act defines a mass layoff as the layoff of either 500 employees or 33 percent of the company's total workforce.

Risk is defined as______. A. probability of X occurrence B. prevention of X avoidance C. prevention of X occurrences D. probability of X consequences

The correct answer is D. The classic definition of Risk is Probability x Consequences.

Once the labor contract is signed, what is the last step in the unionization process? A. conduct a corporate campaign B. sign members up and elect officers C. collect any remaining authorization cards D. employ a grievance procedure to resolve disputes

The correct answer is D. The last step in the unionization process is to employ a grievance procedure to resolve disputes

Janus Inc. is a multimedia marketing firm that internally developed an applicant tracking system (ATS). Which of the following does the company most likely expect the new ATS to do? A. tie the company's employer brand to its product brand B. communicate the company's culture, values, and objectives C. provide an honest picture of the company's work enviornment D. list job openings to the organization's site and online job search engines

The correct answer is D. The other options are what a company's employment brand and EVP should do. One of the functionalities an ATS should offer is to list job openings to the organization's site and online job search engines.

Which job design approach seeks to reduce stress and boredom in an effort to avoid errors or accidents? A. motivational approach B. mechanistic approach C. biological approach D. perceptual-motor approach

The correct answer is D. The perceptual-motor approach to job design seeks to reduce stress and boredom in an effort to avoid errors or accidents.

What is the essential difference between training and development? A. Training costs more than development B. Development applies to an employee's current job; training applies to jobs she might potentially hold in the future C. Training takes longer than development D. Training applies to an employee's current job; development applies to jobs she might potentially hold in the future

The correct answer is D. Training involves job skills for immediate use; development involves the skills needed for both present and future jobs.

Why is it important that applicant testing be subjected to validation? A. because validation shows whether the test was too difficult B. because validation shows whether the applicant has valid chance of passing the test C. because validation shows whether the test was too easy D. because validation shows whether there is a correlation between test performance and job performance

The correct answer is D. Validation shows that there is a demonstrable correlation between how the applicant performs on the test, and how he or she will perform on the job.

Which of the following is not usually a major factor in employer branding? A. unique perks and benefits B. company culture C. Employer Value Proposition (EVP) D. product line

The correct answer is D. Your employer brand is the way your company is perceived by employees and potential job applicants. It is your company's reputation in the job market, and it is the product of many factors, including the perks and benefits it offers, its culture, and the Employer Value Proposition that it defines as part of its branding efforts. But your company's product line is mainly relevant to your customers and your competition; unless it is extraordinarily good or extraordinarily bad, it is unlikely to have a major impact on your employer brand.

What is the Federal Mediation and Conciliation Service (FMCS)?

The federal agency which offers assistance to both parties in reaching an agreement

What is mediation and conciliation?

The process by which a neutral third party is invited to assist in facilitating negotiations

What is the "Excelsior List"?

This "list" includes all eligible bargaining unit employees

What is binding arbitration?

Under this contract provision, the decision must be mandatory on both parties

An organization must set a placement goal when it A. employs a smaller number of women or minorities than is indicated by their availability B. assigns women or minorities to jobs that are not challenging C. is unable to determine the ethnicity of its applicants D. experiences adverse impact in hiring practices

Underutilization is a situation where an organization employs fewer members of a protected class than their availability indicates. For example, if 35% of the paramedics in a labor market are women and a hospital's paramedics are only 10% female, underutilization exists.

What is a zipper clause?

Union contracts often contain this type of clause in which both parties waive the right to demand bargaining on any matter not dealt with in the contract

What is a secondary boycott?

Usually unlawful, this type of union activity is directed at the primary party through actions against a third party

A union may petition for an election after it gets this percent of employees to sign authorization cards

What is 30%?

What is collective bargaining or negotiating?

When both parties enter into discussion with open minds and a desire to arrive at an agreement

Which of the following plans provides tax benefits for both employees and employers, as well as meets guidelines outlined by the Employee Retirement Income Security Act? A. golden parachutes B. omnibus benefit plans C. qualified retirement plans D. non-qualified retirement plans

Which of the following plans provides tax benefits for both employees and employers, as well as meets guidelines outlined by the Employee Retirement Income Security Act?


Set pelajaran terkait

CH 1 LESSON 3 -Computer Programming Languages

View Set

Chapter 5: International Trade Theory

View Set

Raz-M Mother Teresa : Mother to Many

View Set